NCLEX Mental Health 1 of 2

Ace your homework & exams now with Quizwiz!

The nurse notices that a client with obsessive-compulsive disorder washes his hands for long periods each day. How should the nurse respond to this compulsive behavior?

"1. By designating times during which the client can focus on the behavior

The nurse is formulating a short-term goal for a client suffering from a severe obsessive-compulsive disorder (OCD). An appropriately stated short-term goal is that after 1 week, the client will:

"2. participate in a daily exercise group.

Which of the following drugs have been known to be effective in treating obsessive-compulsive disorder (OCD)?

"3. Fluvoxamine (Luvox) and clomipramine (Anafranil)

Which nursing intervention would be most helpful for a client experiencing a panic attack?

"3. Staying with the client and remaining calm, confident, and reassuring

While being escorted to an operating room, a client is extremely anxious and says, "I really don't know what they're going to do to me today. The physician said I have a lump in my breast and that's all I know." Which action is appropriate for the nurse to take?

"4. Notify the physician upon arrival at the operating room.

Two nurses are discussing a client's condition in the elevator. The employer of the mentioned client overhears the conversation and fires the client. The nurses may be liable for which accusation?

"Breach of confidentiality

The physician orders a new medication for a client with generalized anxiety disorder. During medication teaching, which statement or question by the nurse would be most appropriate?

"Do you have any concerns about taking the medication?"

A client in an acute care center lacerates her wrists. She has a history of conflicts and acting out. The client tells the nurse, "I did a good job didn't I?" Which of the following responses would be appropriate?

"What were you feeling before you hurt yourself?"

A client is admitted to an inpatient psychiatric unit for treatment of obsessive-compulsive symptoms. Obsessive-compulsive disorder (OCD) is associated with:

"repetitive thoughts and recurring, irresistible impulses.

(SELECT ALL THAT APPLY) A client is prescribed bupropion (Wellbutrin) to treat depression. The nurse should monitor the client for which adverse reactions associated with bupropion therapy?

(1) Seizures, (2), Anxiety, (3) Insomnia

OCD Interventions

- Don't interrupt the client's compulsive act. - Schedule time for client to compete his ritual (client may perform ritual slowly). - Decrease the time and frequency of the client's rituals. - Distract and substitute self-esteem-building activities. - Provide safety, structure, and activities. - Demonstrate acceptance of the client's feelings.

OCD Client Education

- Teach relaxation techniques. - Teach importance of maintaining medication regimen at home. - Identify triggers that enhance symptoms and develop a plan when symptoms return. - Avoid drug and ETOH use.

A nurse assesses a patient with suspected of having major depression disorder. Which symptom would eliminate the diagnosis? -Maxed out charge cards and exhibits perniscuous behaviors -refuses to interact with others -unable to feel any pleasure 0disheveled and malodorous

-maxed out charge cards and exhibits perniscuous behavior. Depressed mood, loss of interest or pleasure in usual activities and significant weight loss are signs of major depressive disorder.

A 3-year-old child with a history of being abused has blood drawn. The child lies very still and makes no sound during the procedure. Which of the following comments by the nurse would be most appropriate? 1. "It's okay to cry when something hurts." 2. "That really didn't hurt, did it?" 3. "We're mean to hurt you that way, aren't we?" 4. "You were very good not to cry with the needle."

1. It is not normal for a preschooler to be totally passive during a painful procedure. Typically a preschooler reacts to a painful procedure by crying or pulling away because of the fear of pain. However, an abused child may become "immune" to pain and may find that crying can bring on more pain. The child needs to learn that appropriate emotional expression is acceptable. Telling the child that it really didn't hurt is inappropriate because it is untrue. Telling the child that nurses are mean does not build a trusting relationship. Praising the child will reinforce the child's response not to cry, even though it is acceptable to do so.

A client diagnosed with obsessive-compulsive disorder arrives late for an appointment with the nurse at the outpatient clinic. During the interview, he fidgets restlessly, has trouble remembering what topic is being discussed, and says he thinks he is going crazy. Which of the following statements by the nurse best deals with the client's feelings of "going crazy?" 1. "What do you mean when you say you think you're going crazy?" 2. "Most people feel that way occasionally." 3. "I don't know you well enough to judge your mental state." 4. "You sound perfectly sane to me."

1. When the client says he thinks he is "going crazy," it is best for the nurse to ask him what "crazy" means to him. The nurse must have a clear idea of what the client means by his words and actions. Using an open-ended question facilitates client description to help the nurse assess his meaning. The other statements minimize and dismiss the client's concern and do not give him the opportunity to openly discuss his feelings, possibly leading to increased anxiety.

During alprazolam (Xanax) therapy, the nurse should be alert for which dose-related adverse reaction?

1. Ataxia

The nurse refers a client with severe anxiety to a psychiatrist for medication evaluation. The physician is most likely to prescribe which psychotropic drug regimen for this client?

1. Buspirone (BuSpar), 5 mg orally three times per day "

Tricyclics

1. CNS depression 2. Anticholinergic It takes tri weeks for tricyclics to be effective.

A client on an in-patient psychiatric unit is experiencing a flashback. Which intervention takes priority? 1. Maintain and reassure the client of his or her safety and security. 2. Encourage the client to express feelings. 3. Decrease extraneous external stimuli. 4. Use a nonjudgmental and matter-of-fact approach.

1. During a flashback, the client is experiencing severe-to-panic levels of anxiety; the priority nursing intervention is to maintain and reassure the client of his or her safety and security. The client's anxiety needs to decrease before other interventions are attempted. TEST-TAKING HINT: It is important to understand time-wise interventions when dealing with individuals experiencing anxiety. When the client experiences severe-to-panic levels of anxiety during flashbacks, the nurse needs to maintain safety and security until the client's level of anxiety has decreased.

A newly admitted client diagnosed with social phobia has a nursing diagnosis of social isolation R/T fear of ridicule. Which outcome is appropriate for this client? 1. The client will participate in two group activities by day 4. 2. The client will use relaxation techniques to decrease anxiety. 3. The client will verbalize one positive attribute about self by discharge. 4. The client will request buspirone (BuSpar) PRN to attend group by day 2.

1. Expecting the client to participate in a set number of group activities by day 4 directly relates to the stated nursing diagnosis of social isolation and is a measurable outcome that includes a timeframe. TEST-TAKING HINT: To express an appropriate outcome, the statement must be related to the stated problem, be measurable and attainable, and have a timeframe. The test taker can eliminate "2" immediately because there is no timeframe, and then "3" because it does not relate to the stated problem.

After seeking help at an outpatient mental health clinic, a client who was raped while walking her dog is diagnosed with posttraumatic stress disorder (PTSD). Three months later, the client returns to the clinic, complaining of fear, loss of control, and helpless feelings. Which nursing intervention is most appropriate for this client?

1. Exploring the meaning of the traumatic event with the client

The nurse notices that a client with obsessive-compulsive disorder dresses and undresses numerous times each day. Which comment by the nurse would be therapeutic?

1. I saw you change clothes several times today. That must be very tiring.

When working with a group of adult survivors of childhood sexual abuse, dealing with anger and rage is a major focus. Which strategy should the nurse expect to be successful? Select all that apply. 1. Directly confronting the abuser. 2. Using a foam bat while symbolically confronting the abuser. 3. Keeping a journal of memories and feelings. 4. Writing letters to the abusers that are not sent. 5. Writing letters to the adults who did not protect them that are not sent.

2, 3, 4, 5. Using a foam bat while symbolically confronting the abuser, keeping a journal of memories and feelings, and writing letters about the abuse but not sending them are appropriate strategies because they allow anger to be expressed safely. Directly confronting the abuser is likely to result in further harm because the abusers commonly deny the abuse, rationalize about it, or blame the victim.

A client diagnosed with panic attacks is being admitted for the fifth time in 1 year because of hopelessness and helplessness. Which precaution would the nurse plan to implement? 1. Elopement precautions. 2. Suicide precautions. 3. Homicide precautions. 4. Fall precautions.

2. Any client who is exhibiting hopelessness or helplessness needs to be monitored closely for suicide intentions. TEST-TAKING HINT: To answer this question correctly, the test taker should note the words "hopelessness" and "helplessness," which would be indications of suicidal ideations that warrant suicide precautions.

A client is diagnosed with obsessive-compulsive disorder. Which intervention should the nurse include when assisting with development of the plan of care?

2. Giving the client adequate time to perform rituals

(SELECT ALL THAT APPLY) After receiving a referral from the occupational health nurse, a client comes to the mental health clinic with a suspected diagnosis of obsessive-compulsive disorder. The client explains that his compulsion to wash his hands is interfering with his job. Which interventions are appropriate when caring for a client with this disorder?

2. Support the use of appropriate defense mechanisms. 4. Explore the patterns leading to the compulsive behavior. 6. Encourage activities, such as listening to music."

PTSD occurs w/i what frame of the experience?

3 months -in PTSD, the symptoms occur 3 months or more after the trauma, which distinguished PTSD from acute stress disorder which happens right after and last four weeks

When planning interventions for parents who are abusive, the nurse should incorporate knowledge of which factor as a common parental indicator? 1. Lower socioeconomic group. 2. Unemployment. 3. Low self-esteem. 4. Loss of emotional family attachments.

3. Parents who are abusive often suffer from low self-esteem, commonly because of the way they were parented, including not being able to develop trust in caretakers and not being encouraged or offered emotional support by parents. Therefore, the nurse works to bolster the parents' self-esteem. This can be achieved by praising the parents for appropriate parenting. Employment and socioeconomic status are not indicators of abusive parents. Abusive parents usually are attached to their children and do not want to give them up to foster care. Parents who are abusive love their children and feel close to them emotionally.

A client is undergoing treatment for an anxiety disorder. Such a disorder is considered chronic and generalized when excessive anxiety and worry about two or more life circumstances exist for at least:

3. 6 months

A newly admitted client is diagnosed with posttraumatic stress disorder. Which behavioral symptom would the nurse expect to assess? 1. Recurrent, distressing flashbacks. 2. Intense fear, helplessness, and horror. 3. Diminished participation in significant activities. 4. Detachment or estrangement from others.

3. Diminished participation in significant activities is a behavioral symptom of PTSD. TEST-TAKING HINT: To answer this question correctly, the test taker should take note of the keyword "behavioral," which determines the correct answer. All symptoms may be exhibited in PTSD, but only answer choice "3" is a behavioral symptom.

A client enters the crisis unit complaining of increased stress from her studies as a medical student. She states that she has been increasingly anxious for the past month. Her physician prescribes alprazolam (Xanax), 25 mg by mouth three times per day, along with professional counseling. Before administering alprazolam, the nurse reviews the client's medication history. Which drug can produce additive effects when given concomitantly with alprazolam?

3. Diphenhydramine (Benadryl)

A client diagnosed with social phobia has an outcome that states, "Client will voluntarily participate in group activities with peers by day 3." Which would be an appropriate intrapersonal intervention by the nurse to assist the client to achieve this outcome? 1. Offer PRN lorazepam (Ativan) 1 hour before group begins. 2. Attend group with client to assist in decreasing anxiety. 3. Encourage discussion about fears related to socialization. 4. Role-play scenarios that may occur in group to decrease anxiety.

3. Encouraging discussion about fears is an intrapersonal intervention. TEST-TAKING HINT: It is important to understand that interventions are based on theories of causation. In this question, the test taker needs to know that intrapersonal theory relates to feelings or developmental issues. Only "3" deals with client feelings.

A nurse observes a medical student walk into a client's room and begin questioning her about her current health status. The client appears reluctant to respond. How should the nurse intervene?

3. Explain to the client that she has the right to refuse to answer questions asked by the medical student.

A client admitted to the unit is visibly anxious. When collecting data on the client, the nurse would expect to see which cardiovascular effect produced by the sympathetic nervous system?

3. Increased heart rate

A client with obsessive-compulsive disorder and ritualistic behavior must brush the hair back from his forehead 15 times before carrying out any activity. The nurse notices that the client's hair is thinning and the skin on the forehead is irritated — possible effects of this ritual. When planning the client's care, the nurse should assign highest priority to:

3. setting consistent limits on the ritualistic behavior if it harms the client or others.

During the admission data collection, a client with a panic disorder begins to hyperventilate and says, "I'm going to die if I don't get out of here right now!" What is the nurse's best response?

4. ""You're having a panic attack. I'll stay here with you

Victims of sexual assault can experience posttraumatic stress reactions after the attack. Which of the following statements best describes symptoms associated with posttraumatic stress disorder (PTSD)?

4. Flashbacks, recurring dreams, and numbness

While driving home from work, a nurse realizes that she failed to communicate to the oncoming nurse that a client asked for more information about advance directives. Which action would be appropriate for the nurse to take?

4. Phone the nurse caring for the client and inform her of the client's request.

A nursing diagnosis of ineffective coping R/T feelings of loneliness AEB bingeing then purging when alone, is assigned to a client diagnosed with bulimia nervosa. Which is an appropriate outcome related to this nursing diagnosis? A. The client will identify two alternative methods of dealing with isolation by day 3. B. The client will appropriately express angry feelings about lack of control by week 2. C. The client will verbalize two positive self attributes by day 3. D. The client will list five ways that the body reacts to bingeing and purging.

ANS: A The ability to identify alternative methods of dealing with isolation will provide the client with effective coping strategies to use instead of bingeing and purging.

Compliments? with depression

Be careful with these and the depressed patient, it may make them feel worse.

CONVERSION DISORDERS (HYSTERIA) Client Education

Instruct client that stress-relieving activities such as meditation or yoga may help reduce reactions to the events that prompt symptoms of conversion disorder.

What may be influential in the predisposition to PTSD?

Severity of the stressor and availability of support systems.

A patient is diagnosed with schizophrenia. The physician orders Haldol 50 MG BID and Benztropine 1 MG PRN. Which behaviors warrants administration of Benztropine? -Hallucinations -Tremors and shuffling gait - Shouting and screaming

Tremors and shuffling gait

What happens with weight in depression?

Weight gain in mild depression Weight loss in severe depression

symptoms of posttraumatic stress disorder occur within 1 month after the trauma and do not persist longer than 4 weeks

false

in violent families, which normal sage naven may be the most dangerous place for victims

home -the home, which is normally a safe haven of love and protection, may be the most dangerous place for victims

Compulsion is...

recurrent acts

Obsession is...

recurrent thoughts

A client comes to the emergency department while experiencing a panic attack. The nurse should respond to a client having a panic attack by:

staying with the client until the attack subsides

the rate or PTSD- post traumatic stress disorder occurs in rape victims at a rate of -80% -70% -50% -60%

victims of rape have one of the highest rates of PTSD, approximately 70%

A client in the emergency department expresses suicidal ideation and feelings of worthlessness. He has a family history of suicide. The nurse is collecting data on the client. The most important factor to consider is:

whether the client has an active suicide plan and the means to carry it out.

A group of nurses are discussing how food is used in their families and the effects this might have on their ability to work with clients diagnosed with eating disorders. Which of these nurses will probably be most effective with these clients? A. The nurse who understands the importance of three balanced meals a day B. The nurse who permits children to have dessert only after finishing the food on their plate C. The nurse who refuses to engage in power struggles related to food consumption D. The nurse who grew up poor and frequently did not have enough food to eat

ANS: C The nurse who refuses to engage in power struggles related to food consumption will probably be most effective when dealing with clients diagnosed with eating disorders. Because of this attitude the nurse recognizes that the real issues have little to do with food or eating patterns. The nurse will be able to focus on the control issues that precipitated these behaviors.

A nurse assesses a client suspected of having major depressive disorder. Which client symptom would eliminate this diagnosis? A. The client is disheveled and malodorous. B. The client refuses to interact with others. C. The client is unable to feel any pleasure. D. The client has maxed-out charge cards and exhibits promiscuous behaviors.

ANS: D The nurse should assess that a client who has maxed-out credit cards and exhibits promiscuous behavior would be exhibiting manic symptoms. According to the DSM-IV-TR, these symptoms would rule out the diagnosis of major depressive disorder.

A 13-year-old client's father has recently been deployed to Afghanistan. Since deployment, the client has begun to participate in isolative behaviors, truancy, vandalism, and fighting. The pediatric nurse practitioner should identify this behavior with which adjustment disorder? A. An adjustment disorder with anxiety B. An adjustment disorder with disturbance of conduct C. An adjustment disorder with mixed disturbance of emotions and conduct D. An adjustment disorder unspecified

ANS: C The predominant features of an adjustment disorder with mixed disturbance of emotions and conduct include symptoms of anxiety or depression as well as behaviors to include violations of rights of others, truancy, vandalism, and fighting.

The nurse is providing counseling to clients diagnosed with major depressive disorder. The nurse chooses to assess and attempt to modify the negative thought patterns of these clients. The nurse is functioning under which theoretical framework? A. Psychoanalytic theory B. Interpersonal theory C. Cognitive theory D. Behavioral theory

ANS: C When a nurse assesses and attempts to modify negative thought patterns related to depressive symptoms, the nurse is using a cognitive theory framework.

Why are behavior modification programs the treatment of choice for clients diagnosed with eating disorders? A. These programs help clients correct distorted body image. B. These programs address underlying client anger. C. These programs help clients manage uncontrollable behaviors. D. These programs allow clients to maintain control.

ANS: D Behavior modification programs are the treatment of choice for clients diagnosed with eating disorders because these programs allow clients to maintain control. Issues of control are central to the etiology of these disorders. Behavior modification techniques aid in restoring healthy body weight.

Which teaching should the nurse in an employee assistance program provide to an employee who exhibits symptoms of domestic physical abuse? A. Have ready access to a gun and learn how to use it B. Research lawyers who can aid in divorce proceedings C. File charges of assault and battery D. Have ready access to the number of a safe house for battered women

ANS: D The nurse should provide information about safe houses for battered women when working with a client who has symptoms of domestic physical abuse. Many women feel powerless within the abusive relationship and may be staying in the abusive relationship out of fear for their lives.

Treatment of OCD

-Time delay techniques, relaxation techniques -Medications such as SSRIs or TCAs

Bupropion -action on neurotransmitter/receptor -physiological effect -side effect

-inhibits reuptake of NE and dopamine -reduces depression, aid in smoking cessation, decrease in symptoms of ADHD -insomnia, dry mouth, tremor, seizures

A client with panic disorder is taking alprazolam (Xanax) 1 mg P.O. three times daily. The nurse understands that this medication is effective in blocking the symptoms of panic because of its specific action on which of the following neurotransmitters? 1. Gamma-aminobutyrate. 2. Serotonin. 3. Dopamine. 4. Norepinephrine.

1. Alprazolam, a benzodiazepine used on a short-term or temporary basis to treat symptoms of anxiety, increases gamma-aminobutyrate, a major inhibitory neurotransmitter. Because gamma-aminobutyric acid is increased and the reticular activating system is depressed, incoming stimuli are muted and the effects of anxiety are blocked. Alprazolam does not directly target serotonin, dopamine, or norepinephrine.

The client, a veteran of the Vietnam war who has posttraumatic stress disorder, tells the nurse about the horror and mass destruction of war. He states, "I killed all of those people for nothing." Which of the following responses by the nurse is appropriate? 1. "You did what you had to do at that time." 2. "Maybe you didn't kill as many people as you think." 3. "How many people did you kill?" 4. "War is a terrible thing."

1. The nurse states, "You did what you had to do at that time," to help the client evaluate past behavior in the context of the trauma. Clients commonly feel guilty about past behaviors when viewing them in the context of current values. The other statements are inappropriate because they do not help the client to evaluate past behavior in the context of the trauma.

The nurse is caring for a Vietnam War veteran with a history of explosive anger, unemployment, and depression since being discharged from the service. The client reports feeling ashamed of being "weak" and of letting past experiences control thoughts and actions in the present. What is the nurse's best response?

1. "Many people who have been in your situation experience similar emotions and behaviors."

A client is admitted to the psychiatric unit with a diagnosis of conversion disorder. Since witnessing the beating of his wife at gunpoint, he has been unable to move his arms, complaining that they are paralyzed. When planning the client's care, the nurse should focus on:

1. helping the client identify and verbalize feelings about the incident.

A client tells the nurse that she has been raped but has not reported it to the police. After determining whether the client was injured, whether it is still possible to collect evidence, and whether to file a report, the nurse's next priority is to offer which of the following to the client? 1. Legal assistance. 2. Crisis intervention. 3. A rape support group. 4. Medication for disturbed sleep.

2. The experience of rape is a crisis. Crisis intervention services, especially with a rape crisis nurse, are essential to help the client begin dealing with the aftermath of a rape. Legal assistance may be recommended if the client decides to report the rape and only after crisis intervention services have been provided. A rape support group can be helpful later in the recovery process. Medications for sleep disturbance, especially benzodiazepines, should be avoided if possible. Benzodiazepines are potentially addictive and can be used in suicide attempts, especially when consumed with alcohol.

A client with obsessive-compulsive disorder, who was admitted early yesterday morning, must make his bed 22 times before he can have breakfast. Because of his behavior, the client missed having breakfast yesterday with the other clients. Which of the following actions should the nurse institute to help the client be on time for breakfast? 1. Tell the client to make his bed one time only. 2. Wake the client an hour earlier to perform his ritual. 3. Insist that the client stop his activity when it's time for breakfast. 4. Advise the client to have breakfast first before making his bed.

2. The nurse should wake the client an hour earlier to perform his ritual so that he can be on time for breakfast with the other clients. The nurse provides the client with time needed to perform rituals because the client needs to keep his anxiety in check. The nurse should never take away a ritual, because panic will ensue. The nurse should work with the client later to slowly set limits on the frequency of the action.

1. A married female client has been referred to the mental health center because she is depressed. The nurse notices bruises on her upper arms and asks about them. After denying any problems, the client starts to cry and says, "He didn't really mean to hurt me, but I hate for the kids to see this. I'm so worried about them." Which of the following is the most crucial information for the nurse to determine? 1. The type and extent of abuse occurring in the family. 2. The potential of immediate danger to the client and her children. 3. The resources available to the client. 4. Whether the client wants to be separated from her husband.

2. The safety of the client and her children is the most immediate concern. If there is immediate danger, action must be taken to protect them. The other options can be discussed after the client's safety is assured.

A physician's order states to administer lorazepam (Ativan), 20 mg by mouth twice per day, to treat anxiety. How should the nurse proceed?

2. Clarify the order with the prescribing physician because the amount prescribed exceeds the recommended dose.

A client tells the nurse that she has an overwhelming fear of having a heart attack. This client is most likely suffering from which disorder?

2. Panic disorder

Which mental disorders rank among the top ten causes of disability worldwide? (Select all that apply) 1. PTSD 2. Antisocial personality disorder 3. Bipolar affective disorder 4. anxiety disorders 5. Schizophrenia

3. and 5.

during 2001, what percentage of children died form neglect? -15% -48% -50% -33%

33% -1,300 children died from maltreatment, with 33% dying from neglect in 2001

In which situation would the nurse suspect a medical diagnosis of social phobia? 1. A client abuses marijuana daily and avoids social situations because of fear of humiliation. 2. An 8-year-old child isolates from adults because of fear of embarrassment, but has good peer relationships in school. 3. A client diagnosed with Parkinson's disease avoids social situations because of embarrassment regarding tremors and drooling. 4. A college student avoids taking classes that include an oral presentation because of fear of being scrutinized by others.

4. A student who avoids classes because of the fear of being scrutinized by others meets the criteria for a diagnosis of social phobia. TEST-TAKING HINT: The test taker must understand the DSM-IV-TR diagnostic criteria for social phobia to answer this question correctly.

SOMATOFORM DISORDERS

A group of disorders characterized by reports of physical symptoms, with no organic pathology (e.g., a soldier paralyzed during a war, but who has no physical injury). ** SEE CONVERSION DISORDERS **

Dissociative Identity Disorder

AKA multiple personalities An extreme example of dissociative disorder

A college student was sexually assaulted when out on a date. After several weeks of crisis intervention therapy, which client statement should indicate to a nurse that the student is handling this situation in a healthy manner? A. "I know that it was not my fault." B. "My boyfriend has trouble controlling his sexual urges." C. "If I don't put myself in a dating situation, I won't be at risk." D. "Next time I will think twice about wearing a sexy dress."

ANS: A The client who realizes that sexual assault was not her fault is handling the situation in a healthy manner. The nurse should provide nonjudgmental listening and communicate statements that instill trust and validate self-worth.

Which assessment data should a school nurse recognize as signs of physical neglect? A. The child is often absent from school and seems apathetic and tired. B. The child is very insecure and has poor self-esteem. C. The child has multiple bruises on various body parts. D. The child has sophisticated knowledge of sexual behaviors.

ANS: A The nurse should recognize that a child who is often absent from school and seems apathetic and tired might be a victim of neglect. Other indicators of neglect are stealing food or money, lacking medical or dental care, being consistently dirty, lacking sufficient clothing, or stating that there is no one home to provide care.

A nurse is caring for a client who is suspected of having the diagnosis of trichotillomania. What condition must be ruled out prior to a definitive diagnosis of this disorder? A. Bipolar disorder B. Alopecia areata C. Post-traumatic stress disorder D. Body dysmorphic disorder

ANS: B Alopecia areata is a dermatological condition that, according to the DSM-IV diagnostic criteria for trichotillomania, must be ruled out to establish this diagnosis.

A client diagnosed with paranoid personality disorder becomes violent on a unit. Which nursing intervention is most appropriate? A. Provide objective evidence, that violence is unwarranted. B. Initially restrain the client to maintain safety. C. Use clear, calm statements and a confident physical stance. D. Empathize with the client's paranoid perceptions.

ANS: C The most appropriate nursing intervention is to use clear, calm statements and to assume a confident physical stance. A calm attitude avoids escalating the aggressive behavior and provides the client with a feeling of safety and security. It may also be beneficial to have sufficient staff on hand to present a show of strength.

Family dynamics are thought to be a major influence in the development of anorexia nervosa. Which information related to a client's home environment should a nurse associate with the development of this disorder? A. The home environment maintains loose personal boundaries. B. The home environment places an overemphasis on food. C. The home environment is overprotective and demands perfection. D. The home environment condones corporal punishment.

ANS: C The nurse should assess that a home environment that is overprotective and demands perfection may be a major influence in the development of anorexia nervosa. In adolescence, distorted eating patterns may represent a rebellion against controlling and demanding parents.

Which client statement expresses a typical underlying feeling of clients diagnosed with major depressive disorder? A. "It's just a matter of time and I will be well." B. "If I ignore these feelings, they will go away." C. "I can fight these feelings and overcome this disorder." D. "I deserve to feel this way."

ANS: D Hopelessness and helplessness are typical symptoms of clients diagnosed with major depressive disorder. Depressive symptoms are often described as anger turned inward.

A client is admitted to the acute psychiatric care unit after 2 weeks of increasingly erratic behavior. The client has been sleeping poorly, has lost 8 lb (3.6 kg), is poorly groomed, exhibits hyperactivity, and loudly denies the need for hospitalization. Which nursing intervention takes priority for this client?

Administering a sedative as prescribed

CONVERSION DISORDERS (HYSTERIA) Define

Alteration in physical function that is an expression of an unconscious psychological need. Freud suggested that the emotional charge of painful experiences are consciously repressed as a way of managing the pain; the emotional charge is then "converted" into the neurological symptoms. Both somatoform and conversion disorders are used to suppress emotional pain and anxiety.

David, who is 72 years old is of the age at which she may have experienced many losses coming close together. What is this called? -Bereavement overload -isolation -Normal mourning - cultural relativity

Bereavement overload

A client with OCD spends many hours each day washing his hands. The most likely reason he washes his hands so much is that:

It relieves his anxiety.

_______is the malicious or ignorant withholding of physical, emotional, or educational necessities to a dependents well-being

Neglect

Ms. T has been diagnosed with agoraphobia. Which behavior would be most characteristic of this disorder?

She stays in her home for fear of being in a place from which she cannot escape.

________is the repeated and persistent attempts to impose unwanted communication or contact with another person

Stalking

A recently engaged 22-year-old woman loses her fiancé in a drunken driving accident. She complains of difficulty eating, sleeping, and working. Her reaction is considered:

a crisis caused by traumatic stress.

(SELECT ALL THAT APPLY) A physician prescribes clomipramine (Anafranil) for a client diagnosed with obsessive-compulsive disorder (OCD). What instructions should the nurse include when teaching the client about this medication?

"1. Avoid hazardous activities that require alertness or good coordination until adverse central nervous system (CNS) effects are known. 2. Avoid alcohol and other depressants. 3. Use saliva substitutes or sugarless candy or gum to relieve dry mouth. "

The nurse in a psychiatric inpatient unit is caring for a client with obsessive-compulsive disorder. As part of the client's treatment, the psychiatrist orders lorazepam (Ativan), 1 mg by mouth three times per day. During lorazepam therapy, the nurse should remind the client to:

"1. avoid caffeine.

A nurse notices that a client who came to the clinic for treatment of anxiety disorder has a strong body odor. What can the nurse do or say to help this client?

"2. Ask the client basic hygiene questions to determine how frequently he bathes.

The nurse is caring for a client with panic disorder who has difficulty sleeping. Which nursing intervention would best help the client achieve healthy long-term sleeping habits?

"2. Encouraging the use of relaxation exercises

A nurse on the psychiatric unit realizes that she typically fails to administer medications according to schedule. What's the best way for the nurse to improve her medication administration practice?

"2. Evaluate her current practice and devise an improvement plan.

A client admitted to the psychiatric unit for treatment of repeated panic attacks comes to the nurses' station in obvious distress. After observing that the client is short of breath, dizzy, trembling, and nauseated, the nurse should first:

"3. escort the client to a quiet area and suggest using a relaxation exercise that he's been taught.

A client who lost her home and dog in an earthquake tells the admitting nurse at the community health center that she finds it harder and harder to "feel anything." She says she can't concentrate on the simplest tasks, fears losing control, and thinks about the earthquake incessantly. She becomes extremely anxious whenever the earthquake is mentioned and must leave the room if people talk about it. The nurse suspects that she has:

"3. posttraumatic stress disorder (PTSD).

While administering medications to a group of clients admitted with anxiety, a nurse hears someone call for help. The nurse should respond by:

"4. locking the medication cart and responding to the call for help.

A 59-year-old client is scheduled for cardiac catheterization the next morning. His physician prescribed secobarbital sodium (Seconal), 100 mg by mouth at bedtime, for sedation. Before administering the drug, the nurse should know that:

"4. sedatives reduce excitement; hypnotics induce sleep.

A client, age 40, is admitted for a surgical biopsy of a suspicious lump in her left breast. When the nurse comes to take her to surgery, she is tearfully finishing a letter to her two children. She tells the nurse, "I want to leave this for my children in case anything goes wrong today." Which response by the nurse would be most therapeutic?

"In case anything goes wrong? What are your thoughts and feelings right now?"

During the client-teaching session, which instruction should the nurse give to a client receiving alprazolam (Xanax)?

"Inform the physician if you become pregnant or intend to do so."

(SELECT ALL THAT APPLY) The nurse is teaching a client about the antidepressant amitriptyline (Elavil). Which points should she include in her teaching plan?

(1) Smoking may lower the drug level., (2) Avoid prolonged exposure to the sun., (5) Increase fluid and fiber intake to prevent constipation.

SELECT ALL THAT APPLY client suffering posttraumatic stress disorder is prescribed sertraline (Zoloft), 50 mg by mouth once daily. Which actions should the nurse take when administering this drug?

(2) Mix the oral concentrate with 4 oz (120 ml) of water, ginger ale, or lemon-lime soda., (3) Administer the oral solution immediately after dilution., (4) Instruct the client to check with the prescriber or pharmacist before taking over-the-counter preparations., (5) Advise the client to use caution when performing hazardous tasks that require alertness.

PHOBIC DISORDERS Types

(Manifestations depend on the type of phobia.) - Agoraphobia: fear of being places outside the home. - Arachnophobia: fear of spiders. - Social Phobia: **MOST COMMON** (also referred to as Social Anxiety Disorder) irrational fear of embarrassment or ridicule in any social setting or event.

PHOBIC DISORDERS Symptoms

- Feeling of uncontrollable anxiety when exposed to the source of fear. - Attempts made to avoid source and, when exposed, an inability to function normally. - Awareness that fear is unreasonable or exaggerated but client is powerless to control it. - Physical symptoms such as diaphoresis, rapid heart rate, difficulty breathing, and feeling of intense panic and anxiety.

PTSD Symptoms

- Flashback episodes, where the event seems to be happening again and again. - Repeated upsetting memories of the event; repeated nightmares of the event. - Emotional "numbing"; feeling detached; inability to remember important aspects of the trauma. - Having a lack of interest in normal activities. - Avoiding places, people, or thoughts that remind you of the event. - Difficulty concentrating. - Agitation or excitability; insomnia.

OCD Define

- Obsession is a persistent recurring fixed idea or thought that can't be voluntarily removed from consciousness. - Compulsion is an irresistible impulse to perform an action, regardless of it's logic. - (May occur together or separately!) Physiological and biological factors play a role in causing the disorder.

CONVERSION DISORDERS (HYSTERIA) Interventions

- Redirect client away from manifestations. - Encourage client to express feelings. - Teach relaxation and stress-reduction techniques. - Schedule daily activities for the client to decrease the time focused on symptoms.

OCD Symptoms

- Repeated, persistent, and unwanted ideas, thoughts, images, or impulses. - Impulses are involuntary and seem to make no sense. - Obsessions typically intrude when client is attempting to think of or do other things. - Feelings of inferiority, low self-esteem. - Irrational coping to handle guilt.

CONVERSION DISORDERS (HYSTERIA) Symptoms

- Sensory: blindness, deafness, and/or loss of sensation in extremities. - Motor: mutism, ataxia, paralysis. - Visceral: Migraines, dyspnea. - "La Belle Indifference": condition in which the person is unconcerned with symptoms caused by a conversion disorder. A naive, inappropriate lack of emotion or concern for the perceptions by others of one's disability, usually seen in persons with conversion disorder.

PTSD Interventions

- Teach stress reduction techniques. - Identify community support systems. - Encourage client to attend a support group.

PHOBIC DISORDERS Interventions

- Use gradual desensitization experiences. - Employ behavior modification techniques. - Teach relaxation techniques and biofeedback. - Avoid decision making or competitions. - Discuss use of positive coping strategies.

Antianxiety: Buspirone -action on neurotransmitter/receptor -physiological effect -side effect

-5-HT1A agonist, D2 agonist, D2 antagonist -relief of anxiety -nausea, headache, dizziness, restlessness

Other Nursing Implications for Depression

-Sometimes just sitting with the patient is best -interacting with others actually makes them feel better; don't isolate -these people have hard time making simple decisions -as depression lifts, suicide risk goes up -depressed people can have delusions and hallucinations -talk and thoughts are slowed -In mild depression...hypersomnia; in moderate or more, sometimes insomnia

Panic Disorder Notes

-Stay 6 feet away -Simple messages -Have to learn how to stop the anxiety -teach that symptoms should peak within 10 minutes -Teach journaling to manage anxiety -Helps the patient gain insight into the peaks and valleys of anxiety and triggers -Relaxation techniques

Phobia notes

-Usually the object the person is scared of does not present danger -Must have a trusting relationship -Desensitization (gradual exposure to fear) -Don't talk about the phobia a lot -follow-up is the key to successful treatment

Antianxiety: Benzodiazepines -action on neurotransmitter/receptor -physiological effect -side effect

-binds to BZ receptor sites on the GABA-A receptor complex, increases receptor affinity for GABA -relief of anxiety, sedation -dependence, confusion, memory impairment, motor incoordination

Treatment for Dissociative Disorders

-client must process the trauma over time -medications may be used to treat co-existing depression and anxiety

Mania Notes

-continuous high with labile emotions -Delusions of grandeur or persecution -inappropriate dress -spending sprees -no inhibitions -hypersexual -Manipulates a lot...makes them feel powerful and secure -Hallucinations

Crying with depression

-crying spells with mild to moderate depression -no more tears with severe depression

S/S Anorexia

-distorted body image -sees and overweight person in the mirror -won't eat, but preoccupied with food -periods stop -decreased sex development -exercise -lose weight -uses intellectualization as defense mechanism -high achiever, perfectionist

Treatment for Anorexia

-increase weight gradually -monitor exercise routine -teach healthy eating and exercising -allow client input in choosing healthy foods for meals -Limit activity and decisions if weight is low enough to be life threatening

Antipsychotics: Novel: clozapine, olanzepine, aripiprazole, quetiapine, risperdone, ziprasidone, paliperidone, iloperidone, and asenapine -action on neurotransmitter/receptor -physiological effect -side effect

-receptor antagonism of 5-HT1 and 5-HT2, D1-D5, H1, alpha-adrenergic, muscarinic -relief of psychosis, relief of anxiety, and acute mania -potential with some of the drugs for mild EPS, sedation, weight gain, orthrostasis and dizziness, blurred vision, dry mouth, sweating, constipation, urinary retention, tachycardia

Antipsychotics: phenthiazines and haloperidol -action on neurotransmitter/receptor -physiological effect -side effect

-strong D2 receptor blockade, weaker blockade of ACh, H1, alpha-adrenergic and 5-HT2 receptors -relief of psychosis, relief of anxiety, some provide relief from nausea and vomiting and intractable hiccoughs

Dissociative Disorders (signs and symptoms)

-uses dissociation as a coping mechanism to protect self from severe physical and or psychological trauma -may see history of physical or sexual abuse -not common disorder -client or others may be aware of the problem, except that the client may have periods of time or events that he can't remember

A co-manager of a convenience store was taking the daily receipts to the bank when she was robbed at gun point. She did not report the robbery and could not be found for 2 days. In a city 100 miles away, a hotel manager called the police because the woman gave a false name and address. After learning that the robbery was confirmed by the bank cameras, she was admitted to the hospital with a diagnosis of Dissociative Fugue. The nurse should include which of the following in the client's care plan? Select all that apply. 1. Develop trust and rapport to provide safety and support. 2. Rule out possible physical and neurological causes for the fugue. 3. Help the client discuss what she can remember about the trip to the bank. 4. Seclude the client from the other clients because of her lack of memory. 5. Question her repeatedly about the robbery and how she responded. 6. Encourage the client to talk about her feelings about what has been happening.

1, 2, 3, 6. A client experiencing a Dissociate Fugue needs to feel safe and supported as well as evaluated medically and neurologically. Then it is appropriate to discuss what she can remember about the trip to the bank and her feelings about all that has happened to her since then. It is not appropriate to seclude her from others or to apply pressure to get details about the crime at this time. The police and the bank will ask these questions during their investigations.

A client is diagnosed with Generalized Anxiety Disorder (GAD) and given a prescription for venlafaxine (Effexor). Which of the following information should the nurse include in a teaching plan for this client? Select all that apply. 1. Various strategies for reducing anxiety. 2. The benefits and mechanisms of actions of Effexor in treating GAD. 3. How Effexor will eliminate his anxiety at home and work. 4. The management of the common side effects of Effexor. 5. Substituting adaptive coping strategies for maladaptive ones. 6. The positive effects of Effexor being evident in 4 to 5 days.

1, 2, 4, 5. It is appropriate to provide education on medication mechanisms, benefits, and managing side effects. No medication will eliminate all anxiety, so teaching about anxiety reduction and adaptive coping is needed. Effexor is a serotonin-norepinephrine reuptake inhibitor antidepressant and it will take 2 to 4 weeks to feel the effects.

A client diagnosed with Post Traumatic Stress Disorder is readmitted for suicidal thoughts and continued trouble sleeping. She states that when she closes her eyes, she has vivid memories about being awakened at night. "My dad would be on top of me trying to have sex with me. I couldn't breathe." Which of the following suggestions would be appropriate for the nurse to make for the insomnia? Select all that apply. 1. Trying relaxation techniques to help decrease her anxiety before bedtime. 2. Taking the quetiapine (Seroquel) 25 mg as needed as ordered by the physician. 3. Staying in the dayroom and trying to sleep in the recliner chair near staff. 4. Listening to calming music as she tries to fall asleep. 5. Processing the content of her flashbacks no less than hour before bedtime. 6. Leaving her door slightly open to decrease noise during the nightly checks.

1, 2, 4, 6. Relaxation techniques and listening to calming music decrease anxiety and promote sleep. Seroquel is often effective in decreasing nightmare and flashbacks and has a beneficial side effect of drowsiness. Leaving her door slightly open will decrease the noise of making 15 minute checks at night. Staying in the dayroom in a recliner with all the noise and lights is not likely to help. Processing memories an hour or two before bedtime doesn't allow enough time to calm down before sleep.

The nurse is developing a long term care plan for an outpatient client diagnosed with Dissociative Identity Disorder. Which of the following should be included in this plan? Select all that apply. 1. Learning how to manage feelings, especially anger and rage. 2. Joining several outpatient support groups that are process-oriented. 3. Identifying resources to call when there is a risk of suicide or self-mutilation. 4. Selecting a method for alter personalities to communicate with each other, such as journaling. 5. Trying different medicines to find one that eliminates the dissociative process. 6. Helping each alter accept the goal of sharing and integrating all their memories.

1, 3, 4, 6. Managing suicidal thought, urges to self-mutilate and the intense anger are critical safety issues. Then the focus can switch to communication methods for each alter and the integration issues. Process groups can be overwhelming when too much is revealed or when child alters are unable to understand the group content. There are no known medicines to stop the process of dissociating.

A client is taking diazepam (Valium) for generalized anxiety disorder. Which instruction should the nurse give to this client? Select all that apply. 1. To consult with his health care provider before he stops taking the drug. 2. To avoid eating cheese and other tyramine-rich foods. 3. To take the medication on an empty stomach. 4. Not to use alcohol while taking the drug. 5. To stop taking the drug if he experiences swelling of the lips and face and difficulty breathing.

1, 4, 5. The nurse should instruct the client who is taking diazepam to take the medication as prescribed; stopping the medication suddenly can cause withdrawal symptoms. This medication is used for a short term only. The drug dose can be potentiated by alcohol and the client should not drink alcoholic beverages while taking this drug. Swelling of the lips and face and difficulty breathing are signs and symptoms of an allergic reaction. The client should stop taking the drug and seek medical assistance immediately. The client does not need to avoid eating foods containing tyramine; tyramine interacts with monoamine oxidase inhibitors, not Valium. The client can take the medication with food.

A client with obsessive-compulsive disorder reveals that he was late for his appointment "because of my dumb habit. I have to take off my socks and put them back on 41 times! I can't stop until I do it just right." The nurse interprets the client's behavior as most likely representing an effort to obtain which of the following? 1. Relief from anxiety. 2. Control of his thoughts. 3. Attention from others. 4. Safe expression of hostility.

1. A client who is exhibiting compulsive behavior is attempting to control his anxiety. The compulsive behavior is performed to relieve discomfort and to bind or neutralize anxiety. The client must perform the ritual to avoid an extreme increase in tension or anxiety even though the client is aware that the actions are absurd. The repetitive behavior is not an attempt to control thoughts; the obsession or thinking component cannot be controlled. It is not an attention-seeking mechanism or an attempt to express hostility.

Which of the following statements by a client who has been taking buspirone (BuSpar) as prescribed for 2 days indicates the need for further teaching? 1. "This medication will help my tight, aching muscles." 2. "I may not feel better for 7 to 10 days." 3. "The drug does not cause physical dependence." 4. "I can take the medication with food."

1. Buspirone, a nonbenzodiazepine anxiolytic, is particularly effective in treating the cognitive symptoms of anxiety, such as worry, apprehension, difficulty with concentration, and irritability. BuSpar is not effective for the somatic symptoms of anxiety (muscle tension). Therapeutic effects may be experienced in 7 to 10 days, with full effects not occurring for 3 to 4 weeks. This drug is not known to cause physical or psychological dependence. It can be taken with food or small meals to reduce gastrointestinal upset.

A young child who has been sexually abused has difficulty putting feelings into words. Which of the following should the nurse employ with the child? 1. Engaging in play therapy. 2. Role-playing. 3. Giving the child's drawings to the abuser. 4. Reporting the abuse to a prosecutor.

1. The dolls and toys in a play therapy room are useful props to help the child remember situations and reexperience the feelings, acting out the experience with the toys rather than putting the feelings into words. Role-playing without props commonly is more difficult for a child. Although drawing itself can be therapeutic, having the abuser see the pictures is usually threatening for the child. Reporting abuse to authorities is mandatory, but doesn't help the child express feelings.

Which of the following symptom assessments would validate the diagnosis of generalized anxiety disorder? Select all that apply. 1. Excessive worry about items difficult to control. 2. Muscle tension. 3. Hypersomnia. 4. Excessive amounts of energy. 5. Feeling "keyed up" or "on edge."

1. A client diagnosed with generalized anxiety disorder (GAD) would experience excessive worry about items difficult to control. 2. A client diagnosed with GAD would experience muscle tension. 5. A client diagnosed with GAD would experience an increased startle reflex and tension, causing feelings of being "keyed up" or being "on edge." TEST-TAKING HINT: To answer this question correctly, the test taker would need to recognize the signs and symptoms of GAD.

Which of the following would the nurse expect to assess in a client diagnosed with posttraumatic stress disorder? Select all that apply. 1. Dissociative events. 2. Intense fear and helplessness. 3. Excessive attachment and dependence toward others. 4. Full range of affect. 5. Avoidance of activities that are associated with the trauma.

1. A client diagnosed with posttraumatic stress disorder (PTSD) may have dissociative events in which the client feels detached from the situation or feelings. 2. A client diagnosed with PTSD may have intense fear and feelings of helplessness. 5. A client diagnosed with PTSD avoids activities associated with the traumatic event. TEST-TAKING HINT: To answer this question correctly, the test taker must be aware of the different symptoms associated with the diagnosis of PTSD.

A client leaving home for the first time in a year arrives on the psychiatric in-patient unit wearing a surgical mask and white gloves and crying, "The germs in here are going to kill me." Which nursing diagnosis addresses this client's problem? 1. Social isolation R /T fear of germs AEB continually refusing to leave the home. 2. Fear of germs R /T obsessive-compulsive disorder, resulting in dysfunctional isolation. 3. Ineffective coping AEB dysfunctional isolation R /T unrealistic fear of germs. 4. Anxiety R /T the inability to leave home, resulting in dysfunctional fear of germs.

1. According to the North American Nursing Diagnosis Association (NANDA), the nursing diagnosis format must contain three essential components: (1) identification of the health problem, (2) presentation of the etiology (or cause) of the problem, and (3) description of a cluster of signs and symptoms known as "defining characteristics." The correct answer, "1," contains all three components in the correct order: health problem/NANDA stem (social isolation); etiology/cause, or R /T (fear of germs); and signs and symptoms, or AEB (refusing to leave home for the past year). Because this client has been unable to leave home for a year as a result of fear of germs, the client's behaviors meet the defining characteristics of social isolation. TEST-TAKING HINT: To answer this question correctly, the test taker needs to know the components of a correctly stated nursing diagnosis and the order in which these components are written.

A client newly admitted to an in-patient psychiatric unit is diagnosed with obsessivecompulsive disorder. Which correctly stated nursing diagnosis takes priority? 1. Anxiety R /T obsessive thoughts AEB ritualistic behaviors. 2. Powerlessness R /T ritualistic behaviors AEB statements of lack of control. 3. Fear R /T a traumatic event AEB stimulus avoidance. 4. Social isolation R /T increased levels of anxiety AEB not attending groups.

1. Anxiety is the underlying cause of the diagnosis of obsessive compulsive disorder (OCD), therefore, anxiety R/T obsessive thoughts is the priority nursing diagnosis for the client newly admitted for the treatment of this disorder. TEST-TAKING HINT: When the question is asking for a priority, the test taker should consider which client problem would need to be addressed before any other problem can be explored. When anxiety is decreased, social isolation should improve, and feelings about powerlessness can be expressed.

During a panic attack, a client runs to the nurse and reports breathing difficulty, chest pain, and palpitations. The client is pale with his mouth wide open and eyebrows raised. What should the nurse do first?

1. Assist the client to breathe deeply into a paper bag

Because antianxiety agents such as lorazepam (Ativan) can potentiate the effects of other drugs, the nurse should incorporate which instruction in her teaching plan?

1. Avoid mixing antianxiety agents with alcohol or other central nervous system (CNS) depressants

(SELECT AL THAT APPLY) A 54-year-old client diagnosed with generalized anxiety disorder is admitted to the facility. Which therapeutic modalities are typically used to treat this disorder?

1. Biofeedback 2. Buspirone 3. Relaxtion technique

Anxiety is a symptom that can result from which of the following physiological conditions? Select all that apply. 1. Chronic obstructive pulmonary disease. 2. Hyperthyroidism. 3. Hypertension. 4. Diverticulosis. 5. Hypoglycemia.

1. Chronic obstructive pulmonary disease causes shortness of breath. Air deprivation causes anxiety, sometimes to the point of panic. 2. Hyperthyroidism (Graves's disease) involves excess stimulation of the sympathetic nervous system and excessive levels of thyroxine. Anxiety is one of several symptoms brought on by these increases. 5. Marked irritability and anxiety are some of the many symptoms associated with hypoglycemia. TEST-TAKING HINT: To answer this question correctly, the test taker needs to understand that anxiety is manifested by physiological responses.

Which nursing diagnosis reflects the intrapersonal theory of the etiology of obsessivecompulsive disorder? 1. Ineffective coping R /T punitive superego. 2. Ineffective coping R /T active avoidance. 3. Ineffective coping R /T alteration in serotonin. 4. Ineffective coping R /T classic conditioning.

1. Ineffective coping R /T punitive superego reflects an intrapersonal theory of the etiology of obsessive-compulsive disorder (OCD). The punitive superego is a concept contained in Freud's psychosocial theory of personality development. TEST-TAKING HINT: To answer this question correctly, the test taker needs to understand the different theories of the etiology of OCD. The keyword "intrapersonal" should make the test taker look for a concept inherent in this theory, such as "punitive superego."

A hospitalized client diagnosed with posttraumatic stress disorder has a nursing diagnosis of ineffective coping R /T history of rape AEB abusing alcohol. Which is the expected short-term outcome for this client problem? 1. The client will recognize triggers that precipitate alcohol abuse by day 2. 2. The client will attend follow-up weekly therapy sessions after discharge. 3. The client will refrain from self-blame regarding the rape by day 2. 4. The client will be free from injury to self throughout the shift.

1. It is a realistic expectation for a client who copes with previous trauma by abusing alcohol to recognize the triggers that precipitate this behavior. This outcome should be developed mutually early in treatment. TEST-TAKING HINT: It is important to relate outcomes to the stated nursing diagnosis. In this question, the test taker should choose an answer that relates to the nursing diagnosis of ineffective coping. Answer "4" can be eliminated immediately because it does not assist the client in coping more effectively. Also, the test taker must note important words, such as "short-term." Answer "2" can be eliminated immediately because it is a long-term outcome.

When a client experiences a panic attack, which outcome takes priority? 1. The client will remain safe throughout the duration of the panic attack. 2. The client will verbalize an anxiety level less than 2/10. 3. The client will use learned coping mechanisms to decrease anxiety. 4. The client will verbalize the positive effects of exercise by day 2.

1. Remaining safe throughout the duration of the panic attack is the priority outcome for the client. TEST-TAKING HINT: All outcomes must be appropriate for the situation described in the question. In the question, the client is experiencing a panic attack; having the client verbalize the positive effects of exercise would be inappropriate. All outcomes must be client-centered, specific, realistic, positive, and measurable, and contain a timeframe.

A client with obsessive-compulsive disorder may use reaction formation as a defense mechanism to cope with anxiety and stress. What typically occurs in reaction formation?

1. The client assumes an attitude that is the opposite of an impulse that the client harbors.

While in the facility, a client with obsessive-compulsive disorder saves all used medicine cups and paper cups and arranges them in elaborate sculptures in the room. At home, the client saves mail and magazines and makes elaborate paper sculptures from them. Which outcome would indicate successful treatment for this client?

1. The client throws away all disposable cups

A client diagnosed with generalized anxiety disorder complains of feeling out of control and states, "I just can't do this anymore." Which nursing action takes priority at this time? 1. Ask the client, "Are you thinking about harming yourself?" 2. Remove all potentially harmful objects from the milieu. 3. Place the client on a one-to-one observation status. 4. Encourage the client to verbalize feelings during the next group.

1. The nurse should recognize the statement, "I can't do this anymore," as evidence of hopelessness and assess further the potential for suicidal ideations. TEST-TAKING HINT: To answer this question correctly, the test taker should apply the nursing process. Assessment is the first step of this process. The nurse initially must assess a situation before determining appropriate nursing interventions.

Lorazepam (Ativan) is often given along with a neuroleptic agent, such as haloperidol (Haldol). What is the purpose of administering the drugs together?

1. To reduce anxiety and potentiate the sedative action of the neuroleptic

A client diagnosed with posttraumatic stress disorder is close to discharge. Which client statement would indicate that teaching about the psychosocial cause of posttraumatic stress disorder was effective? 1. "I understand that the event I experienced, how I deal with it, and my support system all affect my disease process." 2. "I have learned to avoid stressful situations as a way to decrease emotional pain." 3. "So, natural opioid release during the trauma caused my body to become 'addicted.'" 4. "Because of the trauma, I have a negative perception of the world and feel hopeless."

1. When the client verbalizes understanding of how the experienced event, individual traits, and available support systems affect his or her diagnosis, the client demonstrates a good understanding of the psychosocial cause of posttraumatic stress disorder (PTSD). To answer this question correctly, the test taker should review the different theories as they relate to the causes of different anxiety disorders, including PTSD. Only "1" describes a psychosocial etiology of PTSD.

A client diagnosed with generalized anxiety disorder is prescribed paroxetine (Paxil) 30 mg QHS. Paroxetine is supplied as a 20-mg tablet. The nurse would administer ______tablets.

1.5 tablets

An elderly client is prescribed fluoxetine (Prozac), 40 mg by mouth twice per day, for treatment of depression. The client has difficulty swallowing, so the pharmacy dispenses the oral solution containing 20 mg/5 ml. How many milliliters of solution should the nurse administer to achieve the prescribed dose?

10

A week ago, a tornado destroyed the client's home and seriously injured her husband. The client has been walking around the hospital in a daze without any outward display of emotions. She tells the nurse that she feels like she's going crazy. Which of the following actions should the nurse use first? 1. Explain the effects of stress on the mind and body. 2. Reassure the client that her feelings are typical reactions to serious trauma. 3. Reassure the client that her symptoms are temporary. 4. Acknowledge the unfairness of the client's situation.

2. The nurse initially reassures the client that her feelings and behaviors are typical reactions to serious trauma to help decrease anxiety and maintain self-esteem. Explaining the effects of stress on the body may be helpful later. Telling the client that her symptoms are temporary is less helpful. Acknowledging the unfairness of the client's situation does not address the client's needs at this time.

A preadolescent child is suspected of being sexually abused because he demonstrates the self-destructive behaviors of self-mutilation and attempted suicide. Which common behavior should the nurse also expect to assess? 1. Inability to play. 2. Truancy and running away. 3. Head banging. 4. Over-control of anger.

2. Truancy and running away are common symptoms for young children and adolescents. The stress of the abuse interferes with school success, leading to the avoidance of school. Running away is an effort to escape the abuse and/ or lack of support at home. Rather than an inability to play or a lack of play, play is likely to be aggressive with sexual overtones. Children tend to act out anger rather than control it. Head banging is a behavior typically seen with very young children who are abused.

Which of the following client statements indicates the need for additional teaching about benzodiazepines? 1. "I can't drink alcohol while taking diazepam (Valium)." 2. "I can stop taking the drug anytime I want." 3. "Valium can make me drowsy, so I shouldn't drive for a while." 4. "Valium will help my tight muscles feel better."

2. Valium, like any benzodiazepine, cannot be stopped abruptly. The client must be slowly tapered off of the medication to decrease withdrawal symptoms, which would be similar to withdrawal from alcohol. Alcohol in combination with a benzodiazepine produces an increased central nervous system depressant effect and therefore should be avoided. Valium can cause drowsiness, and the client should be warned about driving until tolerance develops. Valium has muscle relaxant properties and will help tight, tense muscles feel better.

A client with acute stress disorder has avoided feelings of anger toward her rapist and cannot verbally express them. The nurse suggests which of the following activities to assist the client with expressing her feelings? 1. Working on a puzzle. 2. Writing in a journal. 3. Meditating. 4. Listening to music.

2. Writing in a journal can help the client safely express feelings, particularly anger, when the client cannot verbalize them. Safely externalizing anger by writing in a journal helps the client to maintain control over her feelings.

A client with borderline personality disorder tells the nurse, "You're the only nurse who really understands me. The others are mean." The client then asks the nurse for an extra dose of antianxiety medication because of increased anxiety. How should the nurse respond?

2. ""I'll have to discuss your request with the team. Can we talk about how you're feeling right now?""

Which of the following assessment data would support the disorder of acrophobia? 1. A client is fearful of basements because of encountering spiders. 2. A client refuses to go to Europe because of fear of flying. 3. A client is unable to commit to marriage after a 10-year engagement. 4. A client refuses to leave home during stormy weather.

2. Acrophobia is the fear of heights. An individual experiencing acrophobia may be unable to fly because of this fear. TEST-TAKING HINT: To answer this question correctly, the test taker needs to review the definitions of specific commonly diagnosed phobias.

In which situation would benzodiazepines be prescribed appropriately? 1. Long-term treatment of posttraumatic stress disorder, convulsive disorder, and alcohol withdrawal. 2. Short-term treatment of generalized anxiety disorder, alcohol withdrawal, and preoperative sedation. 3. Short-term treatment of obsessive-compulsive disorder, skeletal muscle spasms, and essential hypertension. 4. Long-term treatment of panic disorder, alcohol dependence, and bipolar affective disorder: manic episode.

2. Benzodiazepines are prescribed for shortterm treatment of generalized anxiety disorder and alcohol withdrawal, and can be prescribed during preoperative sedation. TEST-TAKING HINT: The test taker needs to note the words "long-term" and "short-term" in the answers. Benzodiazepines are prescribed in the short-term because of their addictive properties. The test taker must understand that when taking a test, if one part of the answer is incorrect, the whole answer is incorrect, as in answer choice "3."

A client in a psychiatric facility is prescribed escitalopram (Lexapro) for anxiety. She tells the nurse that she has been having "weird dreams" and feelings of wanting to "end it all." What action should the nurse take?

2. Consult a pharmacist to see if these symptoms are adverse effects of the drug.

The nurse is using an intrapersonal approach to assist a client in dealing with survivor's guilt. Which intervention would be appropriate? 1. Encourage the client to attend a survivor's group. 2. Encourage expression of feelings during one-to-one interactions with the nurse. 3. Ask the client to challenge the irrational beliefs associated with the event. 4. Administer regularly scheduled paroxetine (Paxil) to deal with depressive symptoms.

2. Encouraging expressions of feelings during one-to-one interactions with the nurse is an intrapersonal approach to interventions that treat survivor's guilt associated with PTSD. TEST-TAKING HINT: To answer this question correctly, the test taker needs to differentiate various theoretical approaches and which interventions reflect these theories.

A client diagnosed with obsessive-compulsive disorder is newly admitted to an inpatient psychiatric unit. Which cognitive symptom would the nurse expect to assess? 1. Compulsive behaviors that occupy more than 4 hours per day. 2. Excessive worrying about germs and illness. 3. Comorbid abuse of alcohol to decrease anxiety. 4. Excessive sweating and an increase in blood pressure and pulse.

2. Excessive worrying about germs and illness is a cognitive symptom experienced by clients diagnosed with OCD. TEST-TAKING HINT: To answer this question correctly, the test taker must note the keyword "cognitive." Only "2" is a cognitive symptom.

A client recently diagnosed with generalized anxiety disorder is prescribed clonazepam (Klonopin), buspirone (BuSpar), and citalopram (Celexa). Which assessment related to the concurrent use of these medications is most important? 1. Monitor for signs and symptoms of worsening depression and suicidal ideation. 2. Monitor for changes in mental status, diaphoresis, tachycardia, and tremor. 3. Monitor for hyperpyresis, dystonia, and muscle rigidity. 4. Monitor for spasms of face, legs, and neck and for bizarre facial movements.

2. It is important for the nurse to monitor for serotonin syndrome, which occurs when a client takes multiple medications that affect serotonin levels. Symptoms include change in mental status, restlessness, myoclonus, hyperreflexia, tachycardia, labile blood pressure, diaphoresis, shivering, and tremor. TEST-TAKING HINT: To answer this question correctly, the test taker must be familiar with the signs and symptoms of serotonin syndrome and which psychotropic medications affect serotonin, potentially leading to this syndrome.

A client experiencing a panic attack would display which physical symptom? 1. Fear of dying. 2. Sweating and palpitations. 3. Depersonalization. 4. Restlessness and pacing.

2. Sweating and palpitations are physical symptoms of a panic attack. TEST-TAKING HINT: The test taker must note important words in the question, such as "physical symptoms." Although all the answers are actual symptoms a client experiences during a panic attack, only "2" is a physical symptom.

A client diagnosed as having panic disorder with agoraphobia is admitted to the inpatient psychiatric unit. Until her admission, she had been a virtual prisoner in her home for 5 weeks, afraid to go outside even to buy food. When planning care for this client, what is the nurse's overall goal?

2. To help the client function effectively in her environment

A nurse has been providing care to the same group of clients for 4 consecutive days. On day 5, she sees that her assignment has changed, and she is concerned about the continuity of care for these clients. What should the nurse do?

2. Voice her concerns about continuity of care with the charge nurse.

During a shift report, the nurse learns that she will be providing care for a client who's vulnerable to panic attacks. Treatment for panic attacks includes behavioral therapy, supportive psychotherapy, and medication such as

2. antianxiety drugs.

The nurse is collecting data on a client suffering from stress and anxiety. A common physiological response to stress and anxiety is:

2. diarrhea

Initial interventions for the client with acute anxiety include:

2. encouraging the client to verbalize feelings and concerns.

A client with a conversion disorder reports blindness, and ophthalmologic examinations reveal that no physiologic disorder is causing progressive vision loss. The most likely source of this client's reported blindness is:

2. having been forced to watch a loved one's torture.

One of the myths about sexual abuse of young children is that it usually involves physically violent acts. Which of the following behaviors is more likely to be used by the abusers? 1. Tying the child down. 2. Bribery with money. 3. Coercion as a result of the trusting relationship. 4. Asking for the child's consent for sex.

3. Coercion is the most common strategy used because the child commonly trusts the abuser. Tying the child down usually is not necessary. Typically the abusive person can control the child by his or her size and weight alone. Bribery usually is not necessary because the child wants love and affection from the abusive person, not money. Young children are not capable of giving consent for sex before they develop an adult concept of what sex is.

Which of the following points should the nurse include when teaching a client about panic disorder? 1. Staying in the house will eliminate panic attacks. 2. Medication should be taken when symptoms start. 3. Symptoms of a panic attack are time limited and will abate. 4. Maintaining self-control will decrease symptoms of panic.

3. It is important for the nurse to teach the client that the symptoms of a panic attack are time limited and will abate. This helps decrease the client's fear about what is occurring. Clients benefit from learning about their illness, what symptoms to expect, and the helpful use of medication. A simple biologic explanation of the disorder can convince clients to take their medication. Telling the client to stay in the house to eliminate panic attacks is not correct or helpful. Panic attacks can occur "out of the blue," and clients with panic disorder can become agoraphobic because of fear of having a panic attack where help is not available or escape is impossible. Medication should be taken on a scheduled basis to block the symptoms of panic before they start. Taking medication when symptoms start is not helpful. Telling the client to maintain self-control to decrease symptoms of panic is false information because the brain and biochemicals may account for its development. Therefore, the client cannot control when a panic attack will occur.

A client named Jana, with a long history of experiencing Dissociative Identity Disorder, is admitted to the unit after the cuts on her legs were sutured in the Emergency Department. During the admission interview, Jana tearfully states that she does not know what happened to her legs. Then a stronger, alter personality named Jason emerges. Jason states that Jana is useless, weak, and needs to be eliminated completely. The nurse should do which of the following first? 1. Explore Jason's attitudes toward Jana more thoroughly. 2. Place Jana in restraints when Jason emerges. 3. Contract with Jason to tell the nurse when he has the urge to harm Jana and the body they both share. 4. Keep Jana in a stress-free environment so that the stronger Jason does not get a chance to emerge.

3. The No Harm Contract with any destructive alters is essential along with the reminder that the alters share the same body. Later, Jason's attitudes about Jana can be explored in more depth. When alter personalities emerge, their behaviors are not predictable. Restraints could not be placed on the client soon enough. There are no behaviors to justify restraints at this point. Creating a stress-free environment is not possible.

After being discharged from the hospital with acute stress disorder, a client is referred to the outpatient clinic for follow-up. Which of the following is most important for the client to use for continued alleviation of anxiety? 1. Recognizing when she is feeling anxious. 2. Understanding reasons for her anxiety. 3. Using adaptive and palliative methods to reduce anxiety. 4. Describing the situations preceding her feelings of anxiety.

3. The client with anxiety may be able to learn to recognize when she is feeling anxious, understand the reasons for her anxiety, and be able to describe situations that preceded her feelings of anxiety. However, she is likely to continue to experience symptoms unless she has also learned to use adaptive and palliative methods to reduce anxiety.

The nurse notices that a client diagnosed with Major Depression and Social Phobia must get up and move to another area when someone sits next to her. Which of the following actions by the nurse is appropriate? 1. Ignore the client's behavior. 2. Question the client about her avoidance of others. 3. Convey awareness of the client's anxiety about being around others. 4. Tell the other clients to follow the client when she moves away.

3. The nurse conveys empathy and awareness of the client's need to reduce anxiety by showing acceptance and understanding to the client, thereby promoting trust. Ignoring the behavior, questioning the client about her avoidance of others, or telling other clients to follow her when she moves are not therapeutic or appropriate.

When developing the plan of care for a client with acute stress disorder who lost her sister in a boating accident, which of the following should the nurse initiate? 1. Helping the client to evaluate her sister's behavior. 2. Telling the client to avoid details of the accident. 3. Facilitating progressive review of the accident and its consequences. 4. Postponing discussion of the accident until the client brings it up.

3. The nurse should facilitate progressive review of the accident and its consequences to help the client integrate feelings and memories and to begin the grieving process. Helping the client to evaluate her sister's behavior, telling the client to avoid details of the accident, or postponing the discussion of the accident until the client brings it up is not therapeutic and does not facilitate the development of trust in the nurse. Such actions do not facilitate review of the accident, which is necessary to help the client integrate feelings and memories and begin the grieving process.

A client is diagnosed with agoraphobia without panic disorder. Which type of therapy is most effective for this illness? 1. Insight therapy. 2. Group therapy. 3. Behavior therapy. 4. Psychoanalysis.

3. The nurse should suggest behavior therapy, which is most successful for clients with phobias. Systematic desensitization, flooding, exposure, and self-exposure treatments are most therapeutic for clients with phobias. Self-exposure treatment is being increasingly used to avoid frequent therapy sessions. Insight therapy, exploration of the dynamics of the client's personality, is not helpful because the process of anxiety underlies the disorder. Group therapy or psychoanalysis, which deals with repressed, intrapsychic conflicts, is not helpful for the client with phobias because it does not help to manage the underlying anxiety or disorder.

A client with suspected abuse describes her husband as a good man who works hard and provides well for his family. She does not work outside the home and states that she is proud to be a wife and mother just like her own mother. The nurse interprets the family pattern described by the client as best illustrating which of the following as characteristic of abusive families? 1. Tight, impermeable boundaries. 2. Unbalanced power ratio. 3. Role stereotyping. 4. Dysfunctional feeling tone.

3. The traditional and rigid gender roles described by the client are examples of role stereotyping. Impermeable boundaries, unbalanced power ratio, and dysfunctional feeling tone are also common in abusive families.

While a client is taking alprazolam (Xanax), which of the following should the nurse instruct the client to avoid? 1. Chocolate. 2. Cheese. 3. Alcohol. 4. Shellfish.

3. Using alcohol or any central nervous system depressant while taking a benzodiazepine, such as alprazolam, is contraindicated because of additive depressant effects. Ingestion of chocolate, cheese, or shellfish is not problematic.

A woman, age 18, is highly dependent on her parents and fears leaving home to go away to college. Shortly before the fall semester starts, she complains that her legs are paralyzed and is rushed to the emergency department. When physical examination rules out a physical cause for her paralysis, the physician admits her to the psychiatric unit where she is diagnosed with conversion disorder. The client asks the nurse, "Why has this happened to me?" What is the most appropriate response?

3. "Your problem is real but there is no physical basis for it. We'll work on what is going on in your life to find out why it's happened."

Which client would the charge nurse assign to an agency nurse who is new to a psychiatric setting? 1. A client diagnosed with posttraumatic stress disorder currently experiencing flashbacks. 2. A newly admitted client diagnosed with generalized anxiety disorder beginning benzodiazepines for the first time. 3. A client admitted 4 days ago with the diagnosis of algophobia. 4. A newly admitted client with obsessive-compulsive disorder.

3. A client admitted 4 days ago with a diagnosis of algophobia, fear of pain, would be an appropriate assignment for the agency nurse. Of the clients presented, this client would pose the least challenge to a nurse unfamiliar with psychiatric clients. TEST-TAKING HINT: To answer this question correctly, the test taker needs to recognize the complexity of psychiatric diagnoses and understand the ramifications of potentially inappropriate nursing interventions by inexperienced staff members.

The nurse has received evening report. Which client would the nurse need to assess first? 1. A newly admitted client with a history of panic attacks. 2. A client who slept 2 to 3 hours last night because of flashbacks. 3. A client pacing the halls and stating that his anxiety is an 8/10. 4. A client diagnosed with generalized anxiety disorder awaiting discharge.

3. A client pacing the halls and experiencing an increase in anxiety commands immediate assessment. If the nurse does not take action on this assessment, there is a potential for client injury to self or others. TEST-TAKING HINT: When the nurse is prioritizing client assessments, it is important to note which client might be a safety risk. When asked to prioritize, the test taker must review all the situations presented before deciding which one to address first.

A client diagnosed with generalized anxiety disorder is placed on clonazepam (Klonopin) and buspirone (BuSpar). Which client statement indicates teaching has been effective? 1. The client verbalizes that the clonazepam (Klonopin) is to be used for long-term therapy in conjunction with buspirone (BuSpar). 2. The client verbalizes that buspirone (BuSpar) can cause sedation and should be taken at night. 3. The client verbalizes that clonazepam (Klonopin) is to be used short-term until the buspirone (BuSpar) takes full effect. 4. The client verbalizes that tolerance can result with long-term use of buspirone (BuSpar).

3. Clonazepam would be used for shortterm treatment while waiting for the buspirone to take full effect, which can take 4 to 6 weeks. TEST-TAKING HINT: To answer this question correctly, the test taker must note appropriate teaching needs for clients prescribed different classifications of antianxiety medications.

Which assessment data would support a physician's diagnosis of an anxiety disorder in a client? 1. A client experiences severe levels of anxiety in one area of functioning. 2. A client experiences an increased level of anxiety in one area of functioning for a 6-month period. 3. A client experiences increased levels of anxiety that affect functioning in more than one area of life over a 6-month period. 4. A client experiences increased levels of anxiety that affect functioning in at least three areas of life.

3. For a client to be diagnosed with an anxiety disorder, the client must experience symptoms that interfere in a minimum of two areas, such as social, occupational, or other important functioning. These symptoms must be experienced for durations of 6 months or longer. TEST-TAKING HINT: To answer this question correctly, the test taker must understand that specific symptoms must be exhibited and specific timeframes achieved for clients to be diagnosed with anxiety disorders.

A client with a history of generalized anxiety disorder enters the emergency department complaining of restlessness, irritability, and exhaustion. Vital signs are blood pressure 140/90 mm Hg, pulse 96, and respirations 20. Based on this assessed information, which assumption would be correct? 1. The client is exhibiting signs and symptoms of an exacerbation of generalized anxiety disorder. 2. The client's signs and symptoms are due to an underlying medical condition. 3. A physical examination is needed to determine the etiology of the client's problem. 4. The client's anxiolytic dosage needs to be increased.

3. Physical problems should be ruled out before determining a psychological cause for this client's symptoms. TEST-TAKING HINT: The test taker needs to remember that although a client may have a history of a psychiatric illness, a complete, thorough evaluation must be done before assuming exhibited symptoms are related to the psychiatric diagnosis. Many medical conditions generate anxiety as a symptom.

A client seen in an out-patient clinic for ongoing management of panic attacks states, "I have to make myself come to these appointments. It is hard because I don't know when an attack will occur." Which nursing diagnosis takes priority? 1. Ineffective breathing patterns R /T hyperventilation. 2. Impaired spontaneous ventilation R /T panic levels of anxiety. 3. Social isolation R /T fear of spontaneous panic attacks. 4. Knowledge deficit R /T triggers for panic attacks.

3. Social isolation is seen frequently with individuals diagnosed with panic attacks. The client in the question expresses anticipatory fear of unexpected attacks, which affects the client's ability to interact with others. TEST-TAKING HINT: To answer this question correctly, the test taker must link the behaviors presented in the question with the nursing diagnosis that is reflective of these behaviors. The test taker must remember the importance of time-wise interventions. Nursing interventions differ according to the degree of anxiety the client is experiencing. If the client were currently experiencing a panic attack, other interventions would be appropriate.

A client diagnosed with posttraumatic stress disorder has a nursing diagnosis of disturbed sleep patterns R /T nightmares. Which evaluation would indicate that the stated nursing diagnosis was resolved? 1. The client expresses feelings about the nightmares during group. 2. The client asks for PRN trazodone (Desyrel) before bed to fall asleep. 3. The client states that the client feels rested when awakening and denies nightmares. 4. The client avoids napping during the day to help enhance sleep.

3. The client's feeling rested on awakening and denying nightmares are the evaluation data needed to support the fact that the nursing diagnosis of disturbed sleep patterns R/T nightmares has been resolved. TEST-TAKING HINT: To answer this question correctly, the test taker needs to discern evaluation data that indicate problem resolution. Answers "1," "2," and "4" all are interventions to assist in resolving the stated nursing diagnosis, not evaluation data that indicate problem resolution.

Using psychodynamic theory, which intervention would be appropriate for a client diagnosed with panic disorder? 1. Encourage the client to evaluate the power of distorted thinking. 2. Ask the client to include his or her family in scheduled therapy sessions. 3. Discuss the overuse of ego defense mechanisms and their impact on anxiety. 4. Teach the client about the effect of blood lactate level as it relates to the client's panic attacks.

3. The nurse discussing the overuse of ego defense mechanisms illustrates a psychodynamic approach to address the client's behaviors related to panic disorder. TEST-TAKING HINT: When answering this question, the test taker must be able to differentiate among various theoretical perspectives and their related interventions.

Clients diagnosed with obsessive-compulsive disorder commonly use which mechanism? 1. Suppression. 2. Repression. 3. Undoing. 4. Denial.

3. Undoing is a defense mechanism commonly used by individuals diagnosed with OCD. Undoing is used symbolically to negate or cancel out an intolerable previous action or experience. An individual diagnosed with OCD experiencing intolerable anxiety would use the defense mechanism of undoing to undo this anxiety by substituting obsessions or compulsions or both. Other commonly used defense mechanisms are isolation, displacement, and reaction formation. TEST-TAKING HINT: To answer this question correctly, the test taker needs to understand the underlying reasons for the ritualistic behaviors used by individuals diagnosed with OCD.

The nurse is caring for a client experiencing an anxiety attack. Appropriate nursing interventions include:

3. staying with the client and speaking in short sentences.

The mother of a school-aged child tells the nurse that, "For most of the past year my husband was unemployed and I worked a second job. Twice during the year I spanked my son repeatedly when he refused to obey. It has not happened again. Our family is back to normal." After assessing the family, the nurse decides that the child is still at risk for abuse. Which of the following observations best supports this conclusion? 1. The parents say they are taking away privileges when their son refuses to obey. 2. The child has talked about family activities with the nurse. 3. The parent's are less negative toward the nurse. 4. The child wears long sleeve shirts and long pants, even in warm weather.

4. Parental use of nonviolent discipline, the child's talk about what the family is doing and the easing of the parent's negativity toward the school nurse are all signs of progress. Avoidance and wearing clothes inappropriate for the weather implies that the child has something to hide, likely signs of physical abuse.

When obtaining a nursing history from parents who are suspected of abusing their child, which of the following characteristics about the parents should the nurse particularly assess? 1. Attentiveness to the child's needs. 2. Self-blame for the injury to the child. 3. Ability to relate the child's developmental achievements. 4. Difficulty with controlling aggression.

4. Parents of an abused child have difficulty controlling their aggressive behaviors. They may blame the child or others for the injury, may not ask questions about treatment, and may not know developmental information.

The client diagnosed with a fear of eating in public places or in front of other people has finished eating lunch in the dining area in the nurse's presence. Which of the following statements by the nurse should reinforce the client's positive action? 1. "It wasn't so hard, now was it?" 2. "At supper, I hope to see you eat with a group of people." 3. "You must have been hungry today." 4. "It is progress for you to eat in the dining room with me."

4. Saying, "It's a sign of progress to eat in the dining area with me," conveys positive reinforcement and gives the client hope and confidence, thus reinforcing the adaptive behavior. Stating, "It wasn't so hard, now was it," decreases the client's self-worth and minimizes his accomplishment. Stating, "At supper, I hope to see you eat with a group of people," will overwhelm the client and increase anxiety. Stating, "You must have been hungry today," ignores the client's positive behavior and shows the nurse's lack of understanding of the dynamics of the disorder.

116.A newly admitted 20-year-old client, diagnosed with Post Traumatic Stress Disorder (PTSD), reluctantly reveals that she escaped from a satanic cult 2 years ago. The mother has been in the cult since the client was 3 years old and refused to leave with the client. The client says, "Nobody will ever believe the horrible things the men did to me and my mother never stopped them." Which of the following responses is appropriate for the nurse to make? 1. "I'll believe anything you tell me. You can trust me." 2. "I can't understand why your mother didn't protect you. It's not right." 3. "Tell me about the cult. I didn't know there were any near here." 4. "It must be difficult to talk about what happened. I'm willing to listen."

4. Survivors of trauma/ torture have a lot of difficulty with trust and do not readily talk about the horrible events. Therefore, empathy and a willingness to listen without pressuring the client are crucial. Option 1 may or may not be possible and does not convey the empathy. It is sometimes difficult to believe what satanic cults can do to children. Option 2 diverts attention from the client to the mother. Option 3 shows more interest in the cult than the client.

When planning the care for a client who is being abused, which of the following measures is most important to include? 1. Being compassionate and empathetic. 2. Teaching the client about abuse and the cycle of violence. 3. Explaining to the client her personal and legal rights. 4. Helping the client develop a safety plan.

4. The client's safety, including the need to stay alive, is crucial. Therefore, helping the client develop a safety plan is most important to include in the plan of care to ensure the client's safety. Being empathetic, teaching about abuse, and explaining the person's rights are also important after safety is ensured.

A client with posttraumatic stress disorder needs to find new housing and wants to wait for a month before setting another appointment to see the nurse. The nurse interprets this action as which of the following? 1. A method of avoidance. 2. A detriment to progress. 3. The end of treatment. 4. A necessary break in treatment.

4. The nurse judges the client's request for an interruption in treatment as a necessary break in treatment. A "time-out" is common and necessary to enable the client to focus on pressing problems and solutions. It is not necessarily a method of avoidance, a detriment to progress, or the end of treatment. A problem like housing can be very stressful and require all of the client's energy and attention, with none left for the emotional stress of treatment.

The client diagnosed with agoraphobia refuses to walk down the hall to the group room. Which of the following responses by the nurse is appropriate? 1. "I know you can do it." 2. "Try holding onto the wall as you walk." 3. "You can miss group this one time." 4. "I'll walk with you."

4. The nurse should walk with the client to activate adaptive coping for the client experiencing high anxiety and decreased motivation and energy. Stating, "I know you can do it," "Try holding on to the wall," or "You can miss group this one time," maintains the client's avoidance, thus reinforcing the client's behavior, and does not help the client begin to cope with the problem.

When caring for a client who was a victim of a crime, the nurse is aware that recovery from any crime can be a long and difficult process depending on the meaning it has for the client. Which of the following should the nurse establish as a victim's ultimate goal in reconstructing his or her life? 1. Getting through the shock and confusion. 2. Carrying out home and work routines. 3. Resolving grief over any losses. 4. Regaining a sense of security and safety.

4. Ultimately, a victim of a crime needs to move from being a victim to being a survivor. A reasonable sense of safety and security is key to this transition. Getting through the shock and confusion, carrying out home and work routines, and resolving grief over any losses represent steps along the way to becoming a survivor.

In the process of dealing with the intense feelings about being raped, victims commonly verbalize that they were afraid they would be killed during the rape and wish that they had been. The nurse should decide that further counseling is needed if the client voices which of the following? 1. "I didn't fight him, but I guess I did the right thing because I'm alive." 2. "Suicide would be an easy escape from all this pain, but I couldn't do it to myself." 3. "I wish they gave the death penalty to all rapists and other sexual predators." 4. "I get so angry at times that I have to have a couple of drinks before I sleep."

4. Use of alcohol reflects unhealthy coping mechanisms. A client's report of needing alcohol to calm down needs to be addressed. Survival is the most important goal during a rape. The client's acknowledging this indicates that she is aware that she made the right choice. Although suicidal thoughts are common, the statement that suicide is an easy escape but the client would be unable to do it indicates low risk. Fantasies of revenge, such as giving the death penalty to all rapists, are natural reactions and are a problem only if the client intends to carry them out directly.

The nurse teaches an anxious client diagnosed with posttraumatic stress disorder a breathing technique. Which action by the client would indicate that the teaching was successful? 1. The client eliminates anxiety by using the breathing technique. 2. The client performs activities of daily living independently by discharge. 3. The client recognizes signs and symptoms of escalating anxiety. 4. The client maintains a 3/10 anxiety level without medications.

4. A client's ability to maintain an anxiety level of 3/10 without medications indicates that the client is using breathing techniques successfully to reduce anxiety. TEST-TAKING HINT: To answer this question correctly, the test taker should understand that anxiety cannot be eliminated from life. This understanding would eliminate "1" immediately.

During a panic attack, a client hyperventilates, becomes unable to speak, and reports symptoms that mimic those of a heart attack. Which nursing intervention would be best?

4. Accompany the client to his room; remain there and provide instructions in short, simple statements.

A client was admitted to an in-patient psychiatric unit 4 days ago for the treatment of obsessive-compulsive disorder. Which outcome takes priority for this client at this time? 1. The client will use a thought-stopping technique to eliminate obsessive/compulsive behaviors. 2. The client will stop obsessive and/or compulsive behaviors. 3. The client will seek assistance from the staff to decrease obsessive or compulsive behaviors. 4. The client will use one relaxation technique to decrease obsessive or compulsive behaviors.

4. By day 4, it would be realistic to expect the client to use one relaxation technique to decrease obsessive or compulsive behaviors. This would be the current priority outcome. TEST-TAKING HINT: The test taker must recognize the importance of time-wise interventions when establishing outcomes. In the case of clients diagnosed with obsessive-compulsive disorder, expectations on admission vary greatly from outcomes developed closer to discharge.

From a cognitive theory perspective, which is a possible cause of panic disorder? 1. Inability of the ego to intervene when conflict occurs. 2. Abnormal elevations of blood lactate and increased lactate sensitivity. 3. Increased involvement of the neurochemical norepinephrine. 4. Distorted thinking patterns that precede maladaptive behaviors.

4. Distorted thinking patterns that precede maladaptive behaviors relate to the cognitive theory perspective of panic disorder development. TEST-TAKING HINT: The test taker should note important words in the question, such as "cognitive." Although all of the answers are potential causes of panic disorder development, the only answer that is from a cognitive perspective is "4."

(SELECT ALL THAT APPLY) A registered nurse caring for a client with generalized anxiety disorder identifies a nursing diagnosis of Anxiety. The short-term goal identified is: The client will identify his physical, emotional, and behavioral responses to anxiety. Which nursing interventions will help the client achieve this goal?

4. Observe the client for overt signs of anxiety. 5. Help the client connect anxiety with uncomfortable physical, emotional, or behavioral responses. 6. Introduce the client to new strategies for coping with anxiety, such as relaxation techniques and exercise.

A client, who recently delivered a stillborn baby, has a diagnosis of adjustment disorder unspecified. The nurse case manager should expect which client presentation that is characteristic of this diagnosis? A. The client worries continually and appears nervous and jittery. B. The client complains of a depressed mood, is tearful, and feels hopeless. C. The client is belligerent, violates the rights of others, and defaults on legal responsibilities. D. The client complains of many physical ailments, refuses to socialize, and quits her job.

ANS: D The diagnosis of adjustment disorder unspecified is assigned when the maladaptive reaction is not consistent with any of the other categories. Manifestations may include physical complaints, social withdrawal, or work or academic inhibition, without significant depressed or anxious mood.

A newly admitted client diagnosed with posttraumatic stress disorder is exhibiting recurrent flashbacks, nightmares, sleep deprivation, and isolation from others. Which nursing diagnosis takes priority? 1. Posttrauma syndrome R /T a distressing event AEB flashbacks and nightmares. 2. Social isolation R /T anxiety AEB isolating because of fear of flashbacks. 3. Ineffective coping R /T flashbacks AEB alcohol abuse and dependence. 4. Risk for injury R /T exhaustion because of sustained levels of anxiety.

4. Risk for injury is the priority nursing diagnosis for this client. In the question, the client is exhibiting recurrent flashbacks, nightmares, and sleep deprivation that can cause exhaustion and lead to injury. It is important for the nurse to prioritize the nursing diagnosis that addresses safety. TEST-TAKING HINT: When the question asks for a priority, it is important for the test taker to understand that all answer choices may be appropriate statements. Client safety always should be prioritized.

Counselors have been sent to a location that has experienced a natural disaster to assist the population to deal with the devastation. This is an example of __________________ prevention.

4. Sending counselors to a natural disaster site to assist individuals to deal with the devastation is an example of primary prevention. Primary prevention reduces the incidence of mental disorders, such as posttraumatic stress disorder, within the population by helping individuals to cope more effectively with stress early in the grieving process. Primary prevention is extremely important for individuals who experience any traumatic event, such as a rape, war, hurricane, tornado, or school shooting. TEST-TAKING HINT: To answer this question correctly, it is necessary to understand the differences between primary, secondary, and tertiary prevention.

Which of the following statements explains the etiology of obsessive-compulsive disorder (OCD) from a biological theory perspective? 1. Individuals diagnosed with OCD have weak and underdeveloped egos. 2. Obsessive and compulsive behaviors are a conditioned response to a traumatic event. 3. Regression to the pre-Oedipal anal sadistic phase produces the clinical symptoms of OCD. 4. Abnormalities in various regions of the brain have been implicated in the cause of OCD.

4. The belief that abnormalities in various regions of the brain cause OCD is an explanation of OCD etiology from a biological theory perspective. TEST-TAKING HINT: To answer this question correctly, the test taker must understand the different theories of OCD etiology. This question calls for a biological theory perspective, making "4" the only correct choice.

what percentage of staking victims is female?

80%

Physical tolerance and withdrawal symptoms can occur with stimulants. Stimulant withdrawal is characterized by which symptoms?

A violation of confidentiality because she informed the officer that the client wasn't there

A client diagnosed with major depressive disorder was raised in an excessively religiously based household. Which nursing intervention would be most appropriate to address this client's underlying problem? A. Encourage the client to bring into awareness underlying sources of guilt. B. Teach the client that religious beliefs should be put into perspective throughout the life span. C. Confront the client with the irrational nature of the belief system. D. Assist the client to modify his or her belief system in order to improve coping skills.

ANS: A A client raised in an excessively religiously based household maybe at risk for experiencing guilt to the point of accepting liability in situations for which one is not responsible. The client may view himself or herself as evil and deserving of punishment leading to depression. Assisting the client to bring these feelings into awareness allows the client to realistically appraise distorted responsibility and dysfunctional guilt.

A nurse reviews the laboratory data of a client suspected of having major depressive disorder. Which laboratory value would potentially rule out this diagnosis? A. Thyroid-stimulating hormone (TSH) level of 6.2 U/mL B. Potassium (K+) level of 4.2 mEq/L C. Sodium (Na+) level of 140 mEq/L D. Calcium (Ca2+) level of 9.5 mg/dL

ANS: A According to the DSM-IV-TR, symptoms of major depressive disorder cannot be due to the direct physiological effects of a general medical condition (e.g., hypothyroidism). The diagnosis of major depressive disorder may be ruled out if the client's laboratory results indicate a high TSH level which results from a low thyroid function or hypothyroidism. In hypothyroidism, metabolic processes are slowed leading to depressive symptoms.

An advanced practice nurse is counseling a client diagnosed with generalized anxiety disorder. The nurse plans to use activity scheduling to address this client's concerns. What is the purpose of this nursing intervention? A. To identify important areas needing concentration during therapy B. To increase self-esteem and decrease feelings of helplessness C. To modify maladaptive behaviors by the use of role-play D. To divert away from intrusive thoughts and depressive ruminations

ANS: A Activity scheduling is used to identify recurring daily patterns that may need to be addressed in therapy.

When a client on an acute care psychiatric unit demonstrates behaviors and verbalizations indicating a lack of guilt feelings, which nursing intervention would help the client to meet desired outcomes? A. Provide external limits on client behavior. B. Foster discussions of rationales for behavioral change. C. Implement interventions consistently by only one staff member. D. Encourage the client to involve self in care.

ANS: A Because the client, due to a lack of guilt, cannot or will not impose personal limits on maladaptive behaviors, these limits must be delineated and enforced by staff.

A nursing instructor is teaching about the correlation between pathological gambling and abnormalities in the neurotransmitter system. What statement by the nursing student indicates that learning has occurred? A. "Pathological gamblers present with decreased serotonin, increased norepinephrine, and increased dopamine." B. "Pathological gamblers present with increased serotonin, increased norepinephrine, and increased dopamine." C. "Pathological gamblers present with decreased serotonin, decreased norepinephrine, and decreased dopamine." D. "Pathological gamblers present with increased serotonin, decreased norepinephrine, and decreased dopamine."

ANS: A Serotonergic function is linked to behavioral initiation, inhibition, and aggression. Noradrenergic function mediates arousal and detects novel and aversive stimuli. Dopaminergic function is associated with reward and reinforcement mechanisms. Thus, pathological gamblers present with decreased serotonin, increased norepinephrine, and increased dopamine.

Which statement made by an emergency department nurse indicates accurate knowledge of domestic violence? A. "Power and control are central to the dynamic of domestic violence." B. "Poor communication and social isolation are central to the dynamic of domestic violence." C. "Erratic relationships and vulnerability are central to the dynamic of domestic violence." D. "Emotional injury and learned helplessness are central to the dynamic of domestic violence."

ANS: A The nurse accurately states that power and control are central to the dynamic of domestic violence. Battering is defined as a pattern of coercive control founded on physical and/or sexual violence or threat of violence. The typical abuser is very possessive and perceives the victim as a possession.

A nurse observes dental deterioration when assessing a client diagnosed with bulimia nervosa. What explains this assessment finding? A. The emesis produced during purging is acidic and corrodes the tooth enamel. B. Purging causes the depletion of dietary calcium. C. Food is rapidly ingested without proper mastication. D. Poor dental and oral hygiene leads to dental caries.

ANS: A The nurse recognizes that dental deterioration has resulted from the acidic emesis produced during purging that corrodes the tooth enamel. Excessive vomiting may also lead to dehydration and electrolyte imbalance.

A kindergarten student is frequently violent toward other children. A school nurse notices bruises and burns on the child's face and arms. What other symptom should indicate to the nurse that the child might have been physically abused? A. The child shrinks at the approach of adults. B. The child begs or steals food or money. C. The child is frequently absent from school. D. The child is delayed in physical and emotional development.

ANS: A The nurse should determine that a child who shrinks at the approach of adults in addition to having bruises and burns might be a victim of abuse. Whether or not the adult intended to harm the child, maltreatment should be considered.

Which of the following nursing diagnoses could be appropriate for an adult survivor of incest? (Select all that apply.) A. Low self-esteem B. Powerlessness C. Disturbed personal identity D. Knowledge deficit E. Noncompliance

ANS: A, B An adult survivor of incest would most likely have low self-esteem and a sense of powerlessness. Adult survivors of incest are at risk for developing post-traumatic stress disorder, sexual dysfunction, somatization disorders, compulsive sexual behavior disorders, depression, anxiety, eating disorders, and substance abuse disorders. Disturbed personal identity refers to an inability to distinguish between self and nonself and is seen in disorders such as autistic disorders, borderline personality disorders, dissociative disorders, and gender identity disorders.

A client diagnosed with an adjustment disorder asks the nurse, "Tell me about medications that will cure this problem." Which of the following are appropriate nursing replies? (Select all that apply.) A. "Medications can interfere with your ability to find a more permanent problem solution." B. "Medications may mask the real problem at the root of this diagnosis." C. "Adjustment disorders are not commonly treated with medications." D. "Psychoactive drugs carry the potential for physiological and psychological dependence." E. "Psychoactive drugs will be prescribed only if your problems persist for more than 3 months."

ANS: A, B, C, D Adjustment disorder is not commonly treated with medications because of temporary effects, masking the real problem, interfering with finding a permanent solution, and the potential for addiction.

A client has been diagnosed with pathological gambling. The client's family inquires about their brother's behavior that led to this diagnosis. Which of the following information should the clinic nurse provide? (Select all that apply.) A. Your brother has been preoccupied with thoughts about gambling. B. Your brother has been gambling with increased amounts of money to gain excitement. C. Your brother has tried but failed to control his gambling. D. Your brother's gambling is a result of manic behavior. E. Your brother has lied to you about the extent of his gambling.

ANS: A, B, C, E The DSM-IV-TR criteria for the diagnosis of pathological gambling include all and more of the behaviors presented. The gambling behavior cannot be better accounted for by a manic episode.

A client is prescribed phenelzine (Nardil). Which of the following client statements should indicate to a nurse that discharge teaching about this medication has been successful? (Select all that apply.) A. "I'll have to let my surgeon know about this medication before I have my cholecystectomy." B. "Guess I will have to give up my glass of red wine with dinner." C. "I'll have to be very careful about reading food and medication labels." D. "I'm going to miss my caffeinated coffee in the morning." E. "I'll be sure not to stop this medication abruptly."

ANS: A, B, C, E The nurse should evaluate that teaching has been successful when the client states that phenelzine (Nardil) should not be taken in conjunction with the use of alcohol or foods high in tyramine and should not be stopped abruptly. Phenelzine is a monoamine oxidase inhibitor (MAOI) that can have negative interactions with other medications. The client needs to tell other physicians about taking MAOIs due to the risk of drug interactions.

A client is diagnosed with intermittent explosive disorder. The clinic nurse should anticipate teaching about which medication? A. Citalopram (Celexa) B. Risperidone (Risperdal) C. Fluvoxamine (Luvox) D. Isocarboxazid (Marplan)

ANS: B An antipsychotic like Risperdal can be prescribed for intermittent explosive disorder. An antidepressant is not the usual drug of choice for this disorder.

A newly admitted client diagnosed with major depressive disorder states, "I have never considered suicide." Later the client confides to the nurse about plans to end it all by medication overdose. What is the most helpful nursing reply? A. "I'm glad you shared this. There is nothing to worry about. We will handle it together." B. "Bringing this up is a very positive action on your part." C. "We need to talk about the things you have to live for." D. "I think you should consider all your options prior to taking this action."

ANS: B By admitting to the staff a suicide plan, this client has taken responsibility for possible personal actions and expresses trust in the nurse. Therefore, the client may be receptive to continuing a safety plan. Recognition of this achievement reinforces this adaptive behavior.

A nurse working with a client diagnosed with bulimia nervosa asks the client to recall a time in life when food could be consumed without purging. Which is the purpose of this nursing intervention? A. To gain additional information about the progression of the disease process B. To emphasize that the client is capable of consuming food without purging C. To incorporate specific foods into the meal plan to reflect pleasant memories D. To assist the client to become more compliant with the treatment plan

ANS: B By asking the client to recall a time in life when food could be consumed without purging, the nurse is assessing previously successful coping strategies. This information can be used by the client to modify maladaptive behaviors in the present and future.

A client diagnosed with cluster "C" traits sits alone and ignores other's attempts to converse. When ask to join a group the client states, "No thanks." In this situation, which should the nurse assign as an initial nursing diagnosis? A. Fear R/T hospitalization B. Social isolation R/T poor self-esteem C. Risk for suicide R/T to hopelessness D. Powerlessness R/T dependence issues

ANS: B Clients diagnosed with cluster "C" traits are described as anxious and fearful. The DSM-IV-TR divides cluster "C" personality disorders into three categories: avoidant, dependent, and obsessive-compulsive. Anxiety and fear contribute to social isolation.

According to behavioral theory, the treatment of phobic symptoms should involve which action? A. The manipulation of the environment B. The use of desensitization C. The use of family therapy D. The uncovering of past events

ANS: B Systematic desensitization is a technique for assisting individuals to overcome their fear of a phobic stimulus. It is "systematic" in that there is a hierarchy of anxiety-producing events through which the individual progresses during therapy.

Which individual would most likely be diagnosed with intermittent explosive disorder? A. A client diagnosed with antisocial personality disorder who attacks the nursing staff B. A client diagnosed with diabetes mellitus who has a history of multiple severe assaultive acts C. A client diagnosed with schizophrenia who sets fires because of command hallucinations D. A client diagnosed with alcohol dependence who severely beats wife while intoxicated

ANS: B The DSM-IV-TR criteria for the diagnosis of intermittent explosive disorder state that the aggressive episodes are not better accounted for by another mental disorder like antisocial personality disorder or schizophrenia. Also, the aggressive episodes are not due to the direct physiological effect of a substance such as alcohol.

A newly admitted client is diagnosed with major depressive disorder with suicidal ideations. Which would be the priority nursing intervention for this client? A. Teach about the effective of suicide on family dynamics. B. Carefully and unobtrusively observe based on assessed data, at varied intervals around the clock. C. Encourage the client to spend a portion of each day interacting within the milieu. D. Set realistic achievable goals to increase self esteem.

ANS: B The most effective way to interrupt a suicide attempt is to carefully, unobtrusively observe based on assessed data at varied intervals around the clock. If a nurse observes behavior that indicates self-harm, the nurse can intervene to stop the behavior and keep the client safe.

What is the rationale for a nurse to perform a full physical health assessment on a client admitted with a diagnosis of major depressive disorder? A. The attention during the assessment is beneficial in decreasing social isolation. B. Depression can generate somatic symptoms that can mask actual physical disorders. C. Physical health complications are likely to arise from antidepressant therapy. D. Depressed clients avoid addressing physical health and ignore medical problems.

ANS: B The nurse should determine that a client with a diagnosis of major depressive disorder needs a full physical health assessment because depression can generate somatic symptoms that can mask actual physical disorders. Somatization is the process by which psychological needs are expressed in the form of physical symptoms.

The family of a client diagnosed with anorexia nervosa becomes defensive when the treatment team calls for a family meeting. Which is the appropriate nursing reply? A. "Tell me why this family meeting is causing you to be defensive. All clients are required to participate in two family sessions." B. "Eating disorders have been correlated to certain familial patterns; without addressing these, your child's condition will not improve." C. "Family dynamics are not linked to eating disorders. The meeting is to provide your child with family support." D. "Clients diagnosed with anorexia nervosa are part of the family system, and any alteration in family processes needs to be addressed."

ANS: B The nurse should educate the family on the correlation between certain familial patterns and anorexia nervosa. Families engaging in conflict avoidance and struggling with issues of power and control may contribute to the development of this disorder.

When planning care for a client diagnosed with borderline personality disorder, which self-harm behavior should a nurse expect the client to exhibit? A. The use of highly lethal methods to commit suicide B. The use of suicidal gestures to evoke a rescue response from others C. The use of isolation and starvation as suicidal methods D. The use of self-mutilation to decrease endorphins in the body

ANS: B The nurse should expect that a client diagnosed with borderline personality disorder might use suicidal gestures to evoke a rescue response from others. Repetitive, self-mutilative behaviors are common in clients diagnosed with borderline personality disorders. These behaviors are generated by feelings of abandonment following separation from significant others.

Which client is a nurse most likely to admit to an inpatient facility for self-destructive behaviors? A. A client diagnosed with antisocial personality disorder B. A client diagnosed with borderline personality disorder C. A client diagnosed with schizoid personality disorder D. A client diagnosed with paranoid personality disorder

ANS: B The nurse should expect that a client diagnosed with borderline personality disorder would be most likely to be admitted to an inpatient facility for self-destructive behaviors. Clients diagnosed with this disorder often exhibit repetitive, self-mutilative behaviors. Most gestures are designed to evoke a rescue response.

A client who has been taking fluvoxamine (Luvox) without significant improvement asks a nurse, "I heard about something called a monoamine oxidase inhibitor (MAOI). Can't my doctor add that to my medications?" Which is an appropriate nursing reply? A. "This combination of drugs can lead to delirium tremens." B. "A combination of an MAOI and Luvox can lead to a life-threatening hypertensive crisis." C. "That's a good idea. There have been good results with the combination of these two drugs." D. "The only disadvantage would be the exorbitant cost of the MAOI."

ANS: B The nurse should explain to the client that combining an MAOI and Luvox can lead to a life-threatening hypertensive crisis. Symptoms of hypertensive crisis include severe occipital and/or temporal pounding headaches with occasional photophobia, sensations of choking, palpitations, and a feeling of "dread."

An anorexic client states to a nurse, "My father has recently moved back to town." Since that time the client has experienced insomnia, nightmares, and panic attacks that occur nightly. She has never married or dated and lives alone. What should the nurse suspect? A. Possible major depressive disorder B. Possible history of childhood incest C. Possible histrionic personality disorder D. Possible history of childhood physical abuse

ANS: B The nurse should suspect that this client might have a history of childhood incest. Adult survivors of incest are at risk for developing posttraumatic stress disorder, sexual dysfunction, somatization disorders, compulsive sexual behavior disorders, depression, anxiety, eating disorders, and substance abuse disorders.

A client diagnosed with borderline personality disorder brings up a conflict with the staff in a community meeting and develops a following of clients who unreasonably demand modification of unit rules. How can the nursing staff best handle this situation? A. Allow the clients to apply the democratic process when developing unit rules. B. Maintain consistency of care by open communication to avoid staff manipulation. C. Allow the client spokesman to verbalize concerns during a unit staff meeting. D. Maintain unit order by the application of autocratic leadership.

ANS: B The nursing staff can best handle this situation by maintaining consistency of care by open communication to avoid staff manipulation. Clients diagnosed with borderline personality disorder can exhibit negative patterns of interaction such as clinging and distancing, splitting, manipulation, and self-destructive behaviors.

A client is admitted with a diagnosis of depression NOS (not otherwise specified). Which client statement would describe a somatic symptom that can occur with this diagnosis? A. "I am extremely sad, but I don't know why." B. "Sometimes I just don't want to eat because I ache all over." C. "I feel like I can't ever make the right decision." D. "I can't seem to leave the house without someone with me."

ANS: B When a client diagnosed with depression expresses physical complaints, the client is experiencing somatic symptoms. Somatic symptoms occur with depression because of a general slowdown of the entire body reflected in sluggish digestion, constipation, impotence, anorexia, difficulty falling asleep, and a wide variety of other symptoms.

A 75-year-old client diagnosed with a long history of depression is currently on doxepin (Sinequan) 100 mg daily. The client takes a daily diuretic for hypertension and is recovering from the flu. Which nursing diagnosis should the nurse assign highest priority? A. Risk for ineffective thermoregulation R/T anhidrosis B. Risk for constipation R/T excessive fluid loss C. Risk for injury R/T orthostatic hypotension D. Risk for infection R/T suppressed white blood cell count

ANS: C A side effect of Sinequan is orthostatic hypotension. Dehydration due to fluid loss from a combination of diuretic medication and flu symptoms can also contribute to this problem, putting this client at risk for injury R/T orthostatic hypotension.

A nurse would expect a client diagnosed with schizotypal personality disorder to exhibit which characteristic? A. The client keeps to self and has few, if any relationships. B. The client has many brief but intense relationships. C. The client experiences incorrect interpretations of external events. D. The client exhibits lack of tender feelings toward others.

ANS: C Clients who are diagnosed with schizotypal personality disorder experience odd beliefs or magical thinking that influences behavior and is inconsistent with cultural norms. This results in incorrect interpretations of external events.

A client has discovered that her husband is having an affair with a neighbor. During a visit to the neighbor's home, the wife steals the neighbor's diamond ring from the kitchen windowsill. What information would cause a nurse to rule out a diagnosis of kleptomania? A. The wife did not experience a sense of relief when she took the ring. B. The wife did not experience a sense of tension immediately before stealing the ring. C. The stealing was committed to express the wife's anger. D. The ring is desired by the wife for her personal use.

ANS: C The DSM-IV-TR criteria for the diagnosis of kleptomania state that an individual diagnosed with this disorder experiences a sense of tension before committing theft and relief at the time of the theft. The theft cannot be committed as an act of anger or vengeance, and the object stolen cannot be needed for personal use.

A woman comes to an emergency department with a broken nose and multiple bruises after being beaten by her husband. She states, "The beatings have been getting worse, and I'm afraid that next time he might kill me." Which is the appropriate nursing reply? A. "Leopards don't change their spots, and neither will he." B. "There are things you can do to prevent him from losing control." C. "Let's talk about your options so that you don't have to go home." D. "Why don't we call the police so that they can confront your husband with his behavior?"

ANS: C The most appropriate reply by the nurse is to talk with the client about options so that the client does not have to return to the abusive environment. It is essential that clients make decisions independently without the nurse being the "rescuer." Imposing judgments and giving advice is nontherapeutic.

A client diagnosed with antisocial personality disorder comes to a nurses' station at 11:00 p.m. requesting to phone a lawyer to discuss filing for a divorce. The unit rules state that no phone calls are permitted after 10:00 p.m. Which nursing reply is most appropriate? A. "Go ahead and use the phone. I know this pending divorce is stressful." B. "You know better than to break the rules. I'm surprised at you." C. "It is after the 10:00 p.m. phone curfew. You will be able to call tomorrow." D. "The decision to divorce should not be considered until you have had a good night's sleep."

ANS: C The most appropriate response by the staff is to restate the unit rules in a calm, assertive manner. Because of the probability of manipulative behavior in this client population, it is imperative to maintain consistent application of rules.

A depressed client reports to a nurse a history of divorce, job loss, family estrangement, and cocaine abuse. Which theoretical principle best explains the etiology of this client's depressive symptoms? A. According to psychoanalytic theory, depression is a result of anger turned inward. B. According to object-loss theory, depression is a result of abandonment. C. According to learning theory, depression is a result of repeated failures. D. According to cognitive theory, depression is a result of negative perceptions.

ANS: C The nurse should assess that this client's depressive symptoms may have resulted from repeated failures. This assessment was based on the principles of learning theory. Learning theory describes a model of "learned helplessness" in which multiple life failures cause the client to abandon future attempts to succeed.

A highly emotional client presents at an outpatient clinic appointment wearing flamboyant attire, spiked heels, and theatrical makeup. Which personality disorder should a nurse associate with this assessment data? A. Compulsive personality disorder B. Schizotypal personality disorder C. Histrionic personality disorder D. Manic personality disorder

ANS: C The nurse should associate histrionic personality disorder with this assessment data. Individuals diagnosed with histrionic personality disorder tend to be self-dramatizing, attention seeking, overly gregarious, and seductive. They often use manipulation and exhibitionism as a means of gaining attention.

A nursing student asks an emergency department nurse, "Why does a rapist use a weapon during the act of rape?" Which nursing reply is most accurate? A. "A weapon is used to increase the victimizer's security." B. "A weapon is used to inflict physical harm." C. "A weapon is used to terrorize and subdue the victim." D. "A weapon is used to mirror learned family behavior patterns."

ANS: C The nurse should explain that a rapist uses weapons to terrorize and subdue the victim. Rape is the expression of power and dominance by means of sexual violence. Rape can occur over a broad spectrum of experience from violent attack to insistence on sexual intercourse by an acquaintance or spouse.

Which reaction to a compliment from another client should a nurse identify as a typical response from a client diagnosed with avoidant personality disorder? A. Interpreting the compliment as a secret code used to increase personal power B. Feeling the compliment was well deserved C. Being grateful for the compliment but fearing later rejection and humiliation D. Wondering what deep meaning and purpose are attached to the compliment

ANS: C The nurse should identify that a client diagnosed with avoidant personality disorder would be grateful for the comment but would fear later rejection and humiliation. Individuals with avoidant personality disorder are extremely sensitive to rejection and are often awkward and uncomfortable in social situations.

Looking at a slightly bleeding paper cut, the client screams, "Somebody help me, quick! I'm bleeding. Call 911!" A nurse should identify this behavior as characteristic of which personality disorder? A. Schizoid personality disorder B. Obsessive-compulsive personality disorder C. Histrionic personality disorder D. Paranoid personality disorder

ANS: C The nurse should identify this behavior as characteristic of histrionic personality disorder. Individuals diagnosed with this disorder tend to be self-dramatizing, attention seeking, over gregarious, and seductive.

A client is taking clozapine (Clozaril) and complains of a sore throat. This symptom may be an indication of which adverse reaction?

Agranulocytosis

George is diagnosed with Major Depression. She is most likely fixed in which stage of the grief process? -denial -depression -anger -acceptance

Anger

Antisocial Personality

Characteristics 1. Conflicts with society and its rules 2. Unreliable 3. Self-centered 4. Blames others 5. Normal to superior intelligence 6. Poor judgment and insight 7. Unsatisfactory social adjustment 8. Charming Homicidal 9. Difficulty maintaining lasting relationships B. Interventions 1. Set limits 2. Give positive feedback for acceptable behavior 3. Maintain staff communication

Borderline Personality

Characteristics 1. Manipulates others 2. Impulsive 3. Suicidal 4. Poor self-image 5. Bored, trouble being alone 6. Mood swings 7. Anger expressed without control B. Interventions 1. Suicide assessment 2. Set limits 3. Give positive feedback for acceptable behavior 4. Confront inappropriate behavior 5. Encourage expression of feelings, not acting on feelings 6. Don't make decisions for client

Bulimia

Characterized by a. Uncontrollable binge eating b. Self induced vomiting c. Laxative abuse d. Excessive exercise 2. Bulimia differs from anorexia in that the person may maintain a normal weight and is aware that her behavior is abnormal.

A client admitted to the facility continually acts out a preoccupation with hand washing. What term should the nurse use to document this behavior?

Compulsion

An extremely manipulative client is evoking angry feelings in a nurse. Which action should the nurse take first?

Confront the client about his manipulative behavior.

A decrease in norepinephrine may pay a significant role in: o Bipolar disorder o Schizophrenia o Alzheimer's o Depression

Depression

Which nursing intervention is initially most important when restraining a violent client?

Ensuring that the restraints have been applied correctly

A nurse places a client in full leather restraints. How often must the nurse check the client's circulation?

Every 15 minutes

MonAmine Oxidase Inhibitors

Examples and dosage a. Isocarboxazid (Marplan) 10-30 mg b. Phenelzine (Nardil) 45-90 mg c. Tranylcypromine (Parnate) 20-30 mg 2. Combination drugs a. Parphenazine (Etrafon) 16-64 mg b. Amitriptyline (Triavil) 100-300 mg 3. Avoid foods containing tyramine hypertensive crisis a. Aged cheese b. Alcohol c. Fermented foods d. Chocolate e. Yeast f. Raisins g. Bananas 4. Side effects a. Anticholinergic severe b. Central nervous system- sedation c. Many drug interactions

Psychotropic medications that are strong blockers of the D2 receptor are more likely to result in which of the following side effects? -sedation -urinary retention -extrapyramidal symptoms -hypertensive crisis

Extrapyramidal symptoms

Which of therapy is the most appropriate for a client with agoraphobia?

Facing his or her fear in gradual step regression.

Spouse battering, child abuse, elder abuse, and marital rape are examples of ___________violence

Family

Three years ago, Anna's dog Lucky, whom she had for 16 years, was run over by a car and killed. Anna's daughter reports that since that time, Anna has lost weight, rarely leaves her home, and just sits and talks about Lucky. Anna's behavior would be considered maladaptive because: - it has been more than 3 years since Lucky died -her grief is too intense just over the loss of a dog -her grief is interfering with her functioning -people in this culture would not comprehend such behavior

Her grief is interfering with her functioning.

The _____phase in the cycle of violence is a period in which the perpetrator expresses remorse and regret

Honeymoon

Lithium

Initially dose regulated by daily monitoring of blood levels 12 hours after last dose 3. Blood levels a. Therapeutic levels: 0.6-1.2 mEq/L b. Toxic levels: >1.5 mEq/L c. Lethal: >2.5 mEq/L 4. Toxicity 1. Gait disturbances 2. Gastrointestinal 3. Cardiac dysrhythmias 4. Cardiac arrest and death 5. Client education a. It takes 1-4 weeks for therapeutic level to be reached b. Take meds as directed c. Avoid driving until lithium dose stable d. Don't reduce sodium in diet e. Avoid: caffeine, thiazide diuretics, NSAIDs f. Get regular blood tests g. May gain weight h. Notify physician if signs of toxicity

Rape Interventions

Interventions 1. Provide privacy 2. Gather evidence if needed 3. During acute phase a. Establish priorities b. Stay with victim or arrange for someone to do so c. Allow client to wash after assessment and collection of evidence 4. During outward adjustment phase a. Encourage counseling for victim and family 5. During readjustment phase a. Provide list of resources and information and arrange for follow up

Which commonly administered psychiatric medication is prescribed in individualized dosages according to the blood levels of the drug?

Lithium carbonate (Lithane)

Anhedonia

Loss of pleasure in usually pleasurable things

Characteristics of Abuser

Low self-esteem B. Substance abuser C. Projects D. Anxious E. Depressed F. Abused as a child G. Socially isolated H. Impulsive, immature I. Possessive

OCD Treatment

MEDICATIONS: - Antidepressants (Anafranil, Luvox, Prozac, Paxil, Zoloft) THERAPY: - Cog Behavioral Therapy - Psychotherapy

CONVERSION DISORDERS (HYSTERIA) Treatment

MEDICATIONS: - Antidepressants: can be effective with depression, anger, impulsivity, irritability, or hopelessness, which may be associated with personality disorders. - Mood-stabilizers: can be effective to even out mood swings or reduce irritability, impulsivity, and aggression. - Anxiolytics: may be effective to control anxiety, agitation, or insomnia. Can, in some cases, increase impulsivity. - Antipsychotics: may be effective if symptoms include losing touch with reality (psychosis), anxiety, or anger problems. THERAPY: - Psychotherapy - Physical Therapy - Hypnosis

Depression Interventions

Major goal: increase self-esteem 2. Schedule activities of daily living ( structure activities for success) 3. Encourage appropriate amounts of sleep 4. Family or group therapy 5. Unconditional acceptance 6. Encourage expression of feelings

A 49-year-old painter who recently fractured his tibia worries about his finances because he can't work. To treat his anxiety, his physician prescribes buspirone (BuSpar), 5 mg by mouth three times per day. During buspirone therapy, the client should avoid which of the following drugs?

Monoamine oxidase (MAO) inhibitors

Anorexia Nervosa

More common in adolescent or young women 2. Eating disorder in which a person a. Experiences hunger but refuses to eat b. Person has a distorted body image and a self-perception of obesity c, Can lead to starvation and death 3. Often high achievers in school and sports. 4. Fascinated with food. 5. May self induce vomiting 6. Management a. Fluid and electrolyte balance b. Observe client for two hours after each meal. c. Weight assessment on a regular (weekly) basis

Nancy, a depressed client who has been unkempt and untidy for weeks, today comes to group therapy wearing makeup and a clean dress and having washed and combed her hair. Which of the following responses by the nurse is most appropriate? -Nancy, I see you have put on a clean dress and combed your hair -Nancy, you look wonderful today! -Nancy, I'm sure everyone will appreciate that you have cleaned up for the group today -Now that you see how important it is, I hope you will do this everyday

Nancy, I see you have put on a clean dress and comber your hair

OCD

Obsession: Uncontrollable, recurring thoughts B. Compulsion: Ritualistic act done in an attempt to relieve the anxiety related to the thoughts or to make the thoughts go away. C. Interventions 1. Do not interrupt rituals but set limits 2. Set limits 3. Allow time to complete rituals 4. Distract 5. Desensitization 6. Physical protection from repetitive acts usually washing 7. Help client express feelings in appropriate ways 8. Individual and group therapy 9. Anafranil

John, a veteran of the war in Iraq, is diagnosed with PTSD. Which therapy regimen is most appropriate for John?

Paroxetine and group therapy.

Bipolar Disorder

Periods of elation alternating with periods of depression B. Manic: Periods of abnormally elevated mood that are persistent and interfere with functioning C. Assessment findings 1. Quick but superficial wit 2. Flight of ideas 3. Aggressive and argumentative 4. Irritable and hypercritical 5. Increased motor activity

With implosion therapy, a client with phobic anxiety would be:

Presented with massive exposure to a variety of stimuli associated with the phobic object/situation.

Anna, age 72, has been grieving the death of her dog, Lucky, for 3 years. She is not able to take care of her activities of daily living, and wants only to make daily visits to Lucky's grave. Her daughter has likely put off seeking help for Anna because: -women are less likely to seek help for emotional problems than men are -relatives often try to "normalize" the behavior, rather than label it mental illness -she knows that all older people are expected to be a little depressed -she is afraid that the neighbors "will think her mother is crazy"

Relatives often try to "normalize" the behavior, rather than label it a mental illness

An 8-year-old girl and her 5-year-old sister tell the school nurse that their mother frequently yells and spits in their faces when she is mad at them. The nurse hesitates to intervene because she knows the family personally. Which action by the nurse is appropriate?

Report the information to child protective services.

A client is diagnosed with schizophrenia is slow to respond and appears to be listening to unseen others. Which medication administered addresses this positive symptom? -Citalopram -Phenelzine -Risperidone -Sertaline

Risperidone

Depression Characteristics

Sad mood 2. Diminished pleasure 3. Weight loss or gain- change in eating habits and therefore a change in weight 4. Insomnia or hypersomnia 5. Psychomotor agitation or retardation 6. Fatigue 7. Feelings of worthlessness 8. Diminished ability to concentrate 9. Recurrent thoughts of death or suicide

Characteristics of Abused

Same as abuser B. Accepts responsibility for others C. Helpless D. Suicidal at times E. Submissive F. Frightened G. Guilt ridden

Psychotropic medications that block the reuptake of serotonin may result in which of the following side effects? -dry mouth -constipation -blurred vision -sexual dysfunction

Sexual dysfunction

Janet, a psychiatric client diagnosed with Borderline Personality Disorder, has just been hospitalized for threatening suicide. According to Mahler's theory, Janet did not receive the critical "emotional refueling" required during the rapprochement phase of development. That are the consequences of this deficiency? -she has not yet learned to delay gratification -she does not feel guilt about wrong doings to others -she is unable to trust others -she has internalized rage and fears of abandonment

She has internalized rage and fears of abandonment

John, a veteran of the war in Iraq, is diagnosed with PTSD. He experiences a nightmare during his first night in the hospital. He explains to the nurse that he was dreaming about gunfire all around and people being killed. The nurse's most appropriate initial intervention is to:

Stay with John and reassure him of his safety.

John, a veteran of the war in Iraq, is diagnosed with PTSD. he says to the nurse, "I can't figure out why God took my buddy instead of me." From this statement, the nurse assesses that John suffers from:

Survivor's guilt.

Nursing implications for a client taking central nervous system (CNS) stimulants include monitoring the client for which conditions?

Tachycardia, weight loss, and mood swings

A client reports severe pain in the back and joints. Upon reviewing the client's history, the nurse notes a diagnosis of depression and frequent hospitalizations for somatic illnesses. What should the nurse encourage this client to do?

Tell the physician about the pain so that its cause can be determined.

Date rape, also known as_______rape, can occur on a first date or when the two individuals have known eachother for some time

acquaintance

which of the following is not considered a characteristic of violent families? -adequate support systems -alcohol abuse -social isolation -abuse of power and control

adequate support system -having adequate support systems is not a characteristic of a violent family. Abuse of power and control, alcohol abuse, and social isolation are characteristics of violent families

The nurse documents, "The client described her husband's abuse in an emotionless tone and with a flat facial expression." This statement describes the client's:

affect.

which medicaiton classification has been used successfully to treat PTSD?

antidepressants such as Paxil and Zoloft, have been used to treat PTSD

Sedative-hypnotic drugs are indicated for:

anxiety and insomnia.

which of the following is a warning indicator form a caregiver that may indicate elder abuse? -inability to manage finances -failure to keep medical appointments -blaming the elder for his or her illness or limitations -lack of toilet facilities

blaming the elder for his or her illness or limitations -indicators of self-neglect are inability to manage finances, failure to keep mecial appointments, and lack of toilet facilites

which of the following is the most common trait found in abused wives who stay with their husbands? -dependency -jealousy -emotional immaturity -possessiveness

dependency -dependency is the most common trait seen in abused wives who stay with their husbands. -women often cite personal and financial dependency as reasons why they find leaving an abusive relationship extremely difficult

Psychotropic medications that block the acetylcholine receptor may result in which of the following side effects? -dry mouth -sexual dysfunction -nausea -priapism

dry mouth

dissociative amnesia involves episodes of suddenly leaving the home or place of work without any explanation, traveling to another city, and being unable to remember one's past or identity

false

A patient diagnosis with dysthymic disorder. Which symptom should the nurse classify as an affective symptom of this disorder? -Gloomy and pessimistic outlook on life -Low energy level -Difficulty concentration -Social isolation with a focus on self

gloomy, pessimistic outlook on life

which of the following nursing interventions would be the most appropriate to prevent a client from becoming violent? -leaving the client alone until the client can talk about feelings -palce the client in seclusion -helping the client identify and express feelings of anxiety and anger

helping the client identify and express feelings of anxiety and anger -the most appropriate nursing intervention for a client who may become violent is to help him or her identify and express feelings of anxiety and anger. The other intervention would not be the most appropriate for this client situation

A client with antisocial personality disorder smokes where it's prohibited and refuses to follow other unit and facility rules. The client gets others to do his laundry and other personal chores, splits the staff, and will work only with certain nurses. The plan of care for this client should focus primarily on:

isolating the client to decrease contact with easily manipulated clients.

which of the following is a possible indicator of neglect? -hesitance to talk openly -anger -malnourishment not related to a known illness

malnourishment not related to a known illness -malnourishment is a possible indicator of neglect. Helplessness, hesitance to talk openly, and anger are psychological or emotional indicators of abuse

S/S Bulimia

overeat and then vomit teeth decay laxatives, diuretics strict dieter, fasts, exercises binges are alone and in secret normal weight with both they feel like they are in control

a client comes to the emergency department after being attacked and sexually assualted. What is the most accurate nursing diagnosis for this client? -fear -hopelessness -rape-trauma syndrome -anxiety

rape-trauma syndrome -this refers to both the acute and long-term phases experienced by the victim of sexual assault. Specific nursing interventions can be planned based on this diagnosis. The rape victim may experience fear, anxiety, and hopelessness, but these are not specific diagnoses.

A client with obsessive-compulsive disorder tells the nurse that he must check the lock on his apartment door 25 times before leaving for an appointment. The nurse knows that this behavior represents the client's attempt to:

reduce anxiety.

the client is exhibiting intense anger toward the nursing staff after being told that he cannot leave his room. He has previously thrown articles at his family member when he does not get his way. Which of ht e following nursing diagnoses would be the most appropriate to include in the nurisng care plan? -impaired socail interaction -disturbed thought processes -risk for other-directed violence -risk for self-directed violence

risk for other-dirtied violence -a history of violence, threats, violent antisocial behavior, and threatening body language all suggest the nursing diagnosis of risk for other-directed violence

which of the following nursing diagoses has the highest priority for the client diagnosed with PTSD? -ineffective coping -chronic low-self-esteem -risk for self-mutilation -powerlessness

risk of self-mutilation -this is due to the safety issue

in toddler, which injury is most likely the result of child abuse? -a 1-inch forehead laceration -a hematoma on the occipital region of the head -a small, isolated briuse on the right lower extremity -several small, circular burns on the childs back

several small circular burns on the childs back -small circular burns on a childs back are no accidnet and may be from cigarettes. Toddlers are injury prone b/c of their developmental stage, and falls are frequent b/c of their unsteady gait; head injuries are not uncommon. A small area of eccymosis is not suspicious in this age group

exploitation of children is considered which type of abuse? -sexual abuse -neglect -physical abuse -emotional abuse

sexual abuse -sexual abuse can involve exploitation, such as making, promoting, or selling pornography involving minors, and coercion of minors to participate in obscene acts. -neglect is malicious or ignorant withholding of physical, emotional, or educational necessities for the child's well-being. -physical abuse includes burning, biting, or cutting a child -emotional abuse includes verbal assaults, such as blaming, name calling, and using sarcasm

Treatment for Bulimia

sit with clients at meals and observe for 1 hour after allow 30 minutes for meals take focus off the food they may be angry that you have taken the control away family problems are usually the cause self-esteem building is important

the majority of perpetrators of elder abuse include which of the following populations?

spouses -nearly 60% of the perpetrators of elder abuse are spouses, 20% are adult children, and 20% are others such as siblings, grandchildren, and boarders

a parent brings a preschooler to the emergency department for treatment of a dislocated shoulder, which allegedly happened when the child fell down the stairs. Which action should make the nurse supect that the child was abused? -the child cries uncontrollably throughout the examination -the child pulls away from the contact with the doctor -the child does not cry when the shoulder is examined

the child does not cry when the shoulder is examined -a characteristic behavior of abused children is lack of crying when they undergo a painful procedure or are examined by a health care professional. Therefore, the nurse should suspect child abuse. Crying throughout the examination, pulling away from the physician, and not making eye contact with the nurse are normal behaviors for preschoolers

when interviewing the parents of an injured child, which sign is the strongest indicator that child abuse may be a problem?

the injury is not consistent with the history or the childs age -when the childs injuries are inconsistent witht he history given or impossible b/c of the childs age and development stage, the emergency nurse should be suspicious that child abuse is occuring. - the parents may tell different stories cause their perception may be difference regarding what happended. If they change their stroty when different health care workers ask the same question, this is a clue that child abuse may be a problem. -child abuse happens in all socioeconomic groups. -parents may argue and be demanding b/c of the stress of having an injured child

Depression and suicidal behavior are common in surveyors of abuse

true

Intergenerational transmission process suggests that family violence is a pattern of behavior learned form on generation to the next

true

the classification of sodomy as a crime can impede same-sex victims reporting partner abuse?

true

what percentage of victims of intimate violence report that alcohol was involved in the violent incident? -75% -1/4 -2/3 1/2

two thirds of victims of intimate violence report that alcohol was involved in the violent incident

A client who is in a severely abusive relationship is admitted to a psychiatric inpatient unit. The client fears for her life. A staff nurse asks, "Why doesn't she just leave him?" Which is the nursing supervisor's most appropriate reply? A. "These clients don't know life any other way, and change is not an option until they have improved insight." B. "These clients have limited KEY: Cognitive skills and few vocational abilities to be able to make it on their own." C. "These clients often have a lack of financial independence to support themselves and their children, and most have religious beliefs prohibiting divorce and separation." D. "These clients are paralyzed into inaction by a combination of physical threats and a sense of powerlessness."

ANS: D The nursing supervisor is accurate when stating that clients in severely abusive relationships are paralyzed into inaction by a combination of physical threats and a sense of powerlessness. Women often choose to stay with an abusive partner for some of the following reasons: for the children, for financial reasons, fear of retaliation, lack of a support network, religious reasons, and/or hopelessness.

The nurse should warn a client who is taking a benzodiazepine about using which of the following medications in combination with his current medication? 1. Antacids. 2. Acetaminophen (Tylenol). 3. Vitamins. 4. Aspirin.

1. Combining a benzodiazepine with an antacid impairs the absorption rate of the benzodiazepine. Acetaminophen, vitamins, and aspirin are safe to take with a benzodiazepine because no major drug interactions occur.

While shopping at a mall, a woman experiences an episode of extreme terror accompanied by anxiety, tachycardia, trembling, and fear of going crazy. A friend drives her to the emergency department, where a physician rules out physiological causes and refers her to the psychiatric resident on call. To control the client's anxiety, the nurse caring for this client may expect the resident to prescribe:

2. lorazepam (Ativan).

Which of the following observations by the nurse should suggest that a 15-month-old toddler has been abused? 1. The child appears happy when personnel work with him. 2. The child plays alongside others contentedly. 3. The child is underdeveloped for his age. 4. The child sucks his thumb.

3. An almost universal finding in descriptions of abused children is underdevelopment for age. This may be reflected in small physical size or in poor psychosocial development. The child should be evaluated further until a plausible diagnosis can be established. A child who appears happy when personnel work with him is exhibiting normal behavior. Children who are abused often are suspicious of others, especially adults. A child who plays alongside others is exhibiting normal behavior, that of parallel play. A child who sucks his thumb contentedly

A nurse is implementing a one-on-one suicide observation level with a client diagnosed with major depressive disorder. The client states, "I'm feeling a lot better so you can stop watching me. I have taken up too much of your time already." Which is the best nursing reply? A. "I really appreciate your concern but I have been ordered to continue to watch you." B. "Because we are concerned about your safety, we will continue to observe you." C. "I am glad you are feeling better. The treatment team will consider your request." D. "I will forward you request to your psychiatrist because it is his decision."

ANS: B Often suicidal clients resist personal monitoring which impedes the implementation of a suicide plan. A nurse should continually observe a client when risk for suicide is suspected.

A nurse admits an older client who is experiencing memory loss, confused thinking, and apathy. A psychiatrist suspects depression. What is the rationale for performing a mini-mental status exam? A. To rule out bipolar disorder B. To rule out schizophrenia C. To rule out senile dementia D. To rule out a personality disorder

ANS: C A mini-mental status exam should be performed to rule out senile dementia. The elderly are often misdiagnosed with senile dementia when depression is their actual diagnosis. Memory loss, confused thinking, or apathy symptomatic of dementia actually may be the result of depression.

Bipolar Interventions

Set limits 2. Decrease stimuli 3. Basic needs such as finger foods 4. Pace speech 5. Redirect thoughts 6. Avoid arguing 7. Movement activities 8. Distract

(SELECT ALL THAT APPLY) After being examined by the forensic nurse in the emergency department, a rape victim is prepared for discharge. Due to the nature of the attack, this client is at risk for posttraumatic stress disorder (PTSD). Which symptoms are associated with PTSD?

" 1. Recurrent, intrusive recollections or nightmares 3. Sleep disturbances 6. Difficulty concentrating "

The nurse discovers that a client with obsessive-compulsive disorder (OCD) is attempting to resist the compulsion. Based on this finding, the nurse should look for signs of:

4. increased anxiety.

Major Tranquilizers

CNS depression 2. Anticholinergic 3. Parkinson's symptoms 4. Dystonia 5. Photosensitivity 6. Agranulocytosis 7. Neuroleptic malignant syndrome

Which of the following symptoms are seen when a client abruptly stops taking diazepam (Valium)? Select all that apply. 1. Insomnia. 2. Tremor. 3. Delirium. 4. Dry mouth. 5. Lethargy.

Diazepam (Valium) is a benzodiazepine. Benzodiazepines are physiologically and psychologically addictive. If a benzodiazepine is stopped abruptly, a rebound stimulation of the central nervous system occurs, and the client may experience insomnia, increased anxiety, abdominal and muscle cramps, tremors, vomiting, sweating, convulsions, and delirium. 1. Insomnia is correct. 2. Tremor is correct. 3. Delirium is correct. 4. Dry mouth is a side effect of taking benzodiazepines and is not related to stopping the medication abruptly. 5. Lethargy is a side effect of taking benzodiazepines and is not related to stopping the medication abruptly. TEST-TAKING HINT: The test taker must distinguish between benzodiazepine side effectsand symptoms of withdrawal to answer this question correctly.

A nurse is assessing a client who is being abused. The nurse should assess the client for which characteristic? Select all that apply. 1. Assertiveness. 2. Self-blame. 3. Alcohol abuse. 4. Suicidal thoughts. 5. Guilt.

2, 3, 4, 5. The victim of abuse is usually compliant with the spouse and feels guilt, shame, and some responsibility for the battering. Self-blame, substance abuse, and suicidal thoughts and attempts are possible dysfunctional coping methods used by abuse victims. The victim of abuse is not likely to demonstrate assertiveness.

When treating individuals with posttraumatic stress disorder, which variables are included in the recovery environment? 1. Degree of ego strength. 2. Availability of social supports. 3. Severity and duration of the stressor. 4. Amount of control over reoccurrence.

2. Availability of social supports is part of environmental variables. Others include cohesiveness and protectiveness of family and friends, attitudes of society regarding the experience, and cultural and subcultural influences. TEST-TAKING HINT: To answer this question correctly, the test taker needs to understand the following three significant elements in the development of posttraumatic stress disorder: traumatic experience, individual variables, and environmental variables.

Which client statement would demonstrate a common characteristic of Cluster "B" personality disorder? A. "I wish someone would make that decision for me." B. "I built this building by using materials from outer space." C. "I'm afraid to go to group because it is crowded with people." D. "I didn't have the money for the ring, so I just took it."

ANS: D Antisocial personality disorder is included in the Cluster "B" personality disorders. In this disorder there is a pervasive pattern of disregard for and violation of the rights of others.

A welder has been selected as employee of the year. The welder wants to ask for a promotion but is hampered by poor self-esteem. The employee health nurse provides assistance. Which technique should the nurse use to help the employee request the promotion? A. Socratic questioning B. Activity scheduling C. Distraction D. Cognitive rehearsal

ANS: D Cognitive rehearsal allows the employee to uncover potential automatic thoughts in advance of his or her meeting to request a promotion. This allows the employee to develop strategies to modify any dysfunctional thinking.

SSRI

CNS depression 2. Anticholinergic It takes tri weeks for SSRIs to be effective

when the client has a persistnet or recurrent feeling of being detached form his or her mental processes or body, this is documented as which of the following? -dissociative fugue -dissociative identity disorder -dissociative amnesia -depersonalization disorder

depersonalization disorder ---dissociative identity disorder occurs when the client displays two or more distinct identities or personality staes that recurrently take control of his or her behavior. -dissociative fugue occurs when the clients has episodes of suddenly leaving the home or place of work w/o any explanation. -dissociative amnesia occurs when the client cannot remember important personal information. -depersonalization disorder occurs when the client has a persistent or recurrent feeling of being detached from his or her mental processes or body

which type of dissociative disorder involves the clients inability to rememner important personal information

dissociative amnesia --dissociative identity disorder occurs when the client displays two or more distinct identities or personality staes that recurrently take control of his or her behavior. -dissociative fugue occurs when the clients has episodes of suddenly leaving the home or place of work w/o any explanation. -dissociative amnesia occurs when the client cannot remember important personal information. -depersonalization disorder occurs when the client has a persistent or recurrent feeling of being detached from his or her mental processes or body

which of the following is an inaccurate picture of the cycle of abuse that occurs over time? -severity of the injuries worsen -violent episodes are less frequent -violent episodes are more frequent -the period of remorse disappears

violent episodes are less frequent -over time, the violent episodes are more frequent, the period of remorse disappears altogether, and the level of violence and severity of injuries worsen

A client with agoraphobia has been symptom-free for 4 months. Classic signs and symptoms of phobias include:

2. severe anxiety and fear.

Which parental characteristic is least likely to be a risk factor for child abuse? 1. Low self-esteem. 2. History of substance abuse. 3. Inadequate knowledge of normal growth and development patterns. 4. Being a member of a large family.

4. From documented cases of child abuse, a profile has emerged of a high-risk parent as a person who is isolated, impulsive, impatient, and single with low self-esteem, a history of substance abuse, a lack of knowledge about a child's normal growth and development, and multiple life stressors. Just because a parent comes from a large family, there is no increase in the incidence of the parent abusing their own children unless they possess the other risk factors.

A client is diagnosed with dysthymic disorder. Which should a nurse classify as an affective symptom of this disorder? A. Social isolation with a focus on self B. Low energy level C. Difficulty concentrating D. Gloomy and pessimistic outlook on life

ANS: D The nurse should classify a gloomy and pessimistic outlook on life as an affective symptom of dysthymic disorder. Symptoms of depression can be described as alterations in four areas of human functions: affective, behavioral, cognitive, and physiological.

Tricyclic side effects

Anticholinergic b. Cardiovascular c. Photosensitivity d. Two to four weeks to be effective- tricyclics; triweeks

Based on the last question about Anna, Anna's grieving behavior would most likely be considered to be: -delayed -inhibited -prolonged -distorted

Distorted.

A psychiatrist prescribes a monoamine oxidase inhibitor (MAOI) for a client. Which foods should the nurse teach the client to avoid? A. Pepperoni pizza and red wine B. Bagels with cream cheese and tea C. Apple pie and coffee D. Potato chips and diet cola

ANS: A The nurse should instruct the client to avoid pepperoni pizza and red wine. Foods with high tyramine content can induce hypertensive crisis within 2 hours of ingestion. Symptoms of hypertensive crisis include severe occipital and/or temporal pounding headaches with occasional photophobia, sensations of choking, palpitations, and a feeling of "dread."

A client admitted to the psychiatric unit following a suicide attempt is diagnosed with major depressive disorder. Which behavioral symptoms should the nurse expect to assess? A. Anxiety and unconscious anger B. Lack of attention to grooming and hygiene C. Guilt and indecisiveness D. Expressions of poor self-esteem

ANS: B Lack of attention to grooming and hygiene is the only behavioral symptom presented. Depressed clients do not care enough about themselves to participate in grooming and hygiene.

A client diagnosed with schizoaffective disorder is admitted for social skills training. Which information should be taught by the nurse? A. The side effects of medications B. Deep breathing techniques to decrease stress C. How to make eye contact when communicating D. How to be a leader

ANS: C The nurse should plan to teach the client how to make eye contact when communicating. Social skills, such as making eye contact, can assist clients in communicating needs and maintaining connectedness.

During an initial assessment, a client reports the following behaviors: social inhibition, hypersensitivity to negative evaluation, fear of criticism, and social ineptitude. The nurse suspects which of the following personality disorders?

Avoidant

A patient diagnosed with schizophrenia is prescribed Clozapine (Clozaril). Which symptoms present should the nurse intervene immediately? -Sore throat, fever, malaise -Akathisia and hypersalivation -Akinesia and insomnia -Dry mouth and urinary retention

Sore throat, fever, malaise

A client with borderline personality disorder dramatically expresses feelings about each nurse on the staff, stating that only one nurse is understanding and trustworthy — the nurse the client is talking to at the time. This client is demonstrating which behavior?

Splitting

what percentage of women can expect to be victim of an ongoing unwanted pursuit from stalking? -25% -50% -10% -40%

25%

After a client reveals a history of childhood sexual abuse, the nurse should ask which of the following questions first ? 1. "What other forms of abuse did you experience?" 2. "How long did the abuse go on?" 3. "Was there a time when you did not remember the abuse?" 4. "Does your abuser still have contact with young children?"

4. The safety of other children is a primary concern. It is critical to know whether other children are at risk for being sexually abused by the same perpetrator. Asking about other forms of abuse, how long the abuse went on, and if the victim did not remember the abuse are important questions after the safety of other children is determined.

An isolative client was admitted 4 days ago with a diagnosis of major depressive disorder. Which nursing statement would best motivate this client to attend a therapeutic group being held in the milieu? A. "We'll go to the day room when you are ready for group." B. "I'll walk with you to the day room. Group is about to start." C. "It must be difficult for you to attend group when you feel so bad." D. "Let me tell you about the benefits of attending this group."

ANS: B A client diagnosed with major depressive disorder exhibits little to no motivation and must be firmly directed by staff to participate in therapy. It is difficult for a severely depressed client to make decisions, and this function must be temporarily assumed by the staff.

A morbidly obese client is prescribed an anorexiant medication. The nurse should prepare to teach the client about which medication? A. Diazepam (Valium) B. Dexfenfluramine (Redux) C. Sibutramine (Meridia) D. Pemoline (Cylert)

ANS: C The nurse should teach the client that sibutramine (Meridia) is an anorexiant medication prescribed for morbidly obese clients. The mechanism of action in the control of appetite appears to occur by inhibiting the neutotransmitters serotonin and norepinephrine. Withdrawal from anorexiants can result in rebound weight gain, lethargy, and depression.

The initial care plan for a client with OCD who washes her hands obsessively would include which nursing intervention?

Sets limitations on the amount of time the client may engage in the ritualistic behavior.

which type of male rapist impulsively ices his victims as objects for gratification? -inadequate men -sexual sadists -exploitive predators -men for who anger is displaced

exploitive predators -exploitive predators impulsively use their victims as objects for gratification. -Sexual sadists are aroused by the pain of their victim -inadequate men believe that no woman would voluntarily have sexual relations with them and are obsessed with fantasies about sex

Before eating a meal, a client with obsessive-compulsive disorder (OCD) must wash his hands for 18 minutes, comb his hair 444 strokes, and switch the bathroom light on and off 44 times. What is the most appropriate goal of care for this client?

"Systematically decrease the number of repetitions of rituals and the amount of time spent performing them.

A client is brought to the emergency department by his brother. The client is perspiring profusely, breathing rapidly, and complaining of dizziness and palpitations. Problems of a cardiovascular nature are ruled out, and the client's diagnosis is tentatively listed as a panic attack. After the symptoms pass, the client states, "I thought I was going to die." Which of the following responses by the nurse is best? 1. "It was very frightening for you." 2. "We would not have let you die." 3. "I would have felt the same way." 4. "But you're okay now."

1. The nurse responds with the statement, "It was very frightening for you," to express empathy, thus acknowledging the client's discomfort and accepting his feelings. The nurse conveys respect and validates the client's self-worth. The other statements do not focus on the client's underlying feelings, convey active listening, or promote trust.

Which of the following medications can be used to treat clients with anxiety disorders? Select all that apply. 1. Clonidine hydrochloride (Catapres). 2. Fluvoxamine maleate (Luvox). 3. Buspirone (BuSpar). 4. Alprazolam (Xanax). 5. Haloperidol (Haldol).

1. Clonidine hydrochloride (Catapres) is used in the treatment of panic disorders and generalized anxiety disorder. 2. Fluvoxamine maleate (Luvox) is used in the treatment of obsessive-compulsive disorder. 3. Buspirone (BuSpar) is used in the treatment of panic disorders and generalized anxiety disorders. 4. Alprazolam (Xanax), a benzodiazepine, is used for the short-term treatment of anxiety disorders. TEST-TAKING HINT: To answer this question correctly, the test taker needs to understand that many medications are used off-label to treat anxiety disorders.

A 10-year-old client diagnosed with nightmare disorder is admitted to an in-patient psychiatric unit. Which of the following interventions would be appropriate for this client's problem? Select all that apply. 1. Involving the family in therapy to decrease stress within the family. 2. Using phototherapy to assist the client to adapt to changes in sleep. 3. Administering medications such as tricyclic antidepressants or low-dose benzodiazepines or both. 4. Giving central nervous system stimulants, such as amphetamines. 5. Using relaxation therapy, such as meditation and deep breathing techniques, to assist the client in falling asleep.

1. Family stress can occur as the result of repeated client nightmares. This stress within the family may exacerbate the client's problem and hamper any effective treatment. Involving the family in therapy to relieve obvious stress would be an appropriate intervention to assist in the treatment of clients diagnosed with a nightmare disorder. 3. Administering medications such as tricyclic antidepressants or low-dose benzodiazepines or both is an appropriate intervention for clients diagnosed with a parasomnia disorder, such as a nightmare disorder. 5. Relaxation therapy, such as meditation and deep breathing techniques, would be appropriate for clients diagnosed with a nightmare disorder to assist in falling back to sleep after the nightmare occurs. TEST-TAKING HINT: To answer this question correctly, the test taker must be able first to understand the manifestation of a nightmare disorder and then to choose the interventions that would address these manifestations effectively.

A client diagnosed with generalized anxiety disorder has a nursing diagnosis of panic anxiety R/T altered perceptions. Which of the following short-term outcomes is most appropriate for this client? 1. The client will be able to intervene before reaching panic levels of anxiety by discharge. 2. The client will verbalize decreased levels of anxiety by day 2. 3. The client will take control of life situations by using problem-solving methods effectively. 4. The client will voluntarily participate in group therapy activities by discharge.

1. The client's being able to intervene before reaching panic levels of anxiety by discharge is measurable, relates to the stated nursing diagnosis, has a timeframe, and is an appropriate short-term outcome for this client. TEST-TAKING HINT: When evaluating outcomes, the test taker must make sure that the outcome is specific to the client's need, is realistic, is measurable, and contains a reasonable timeframe. If any of these components is missing, the outcome is incorrectly written and can be eliminated.

The nurse is using a cognitive intervention to decrease anxiety during a client's panic attack. Which statement by the client would indicate that the intervention has been successful? 1. "I reminded myself that the panic attack would end soon, and it helped." 2. "I paced the halls until I felt my anxiety was under control." 3. "I felt my anxiety increase, so I took lorazepam (Ativan) to decrease it." 4. "Thank you for staying with me. It helped to know staff was there."

1. This statement is an indication that the cognitive intervention was successful. By remembering that panic attacks are self-limiting, the client is applying the information gained from the nurse's cognitive intervention. TEST-TAKING HINT: To answer this question correctly, the test taker needs to understand which interventions support which theories of causation. When looking for a "cognitive" intervention, the test taker must remember that the theory involves thought processes.

A client newly admitted to an in-patient psychiatric unit is diagnosed with obsessivecompulsive disorder. Which behavioral symptom would the nurse expect to assess? 1. The client uses excessive hand washing to relieve anxiety. 2. The client rates anxiety at 8/10. 3. The client uses breathing techniques to decrease anxiety. 4. The client exhibits diaphoresis and tachycardia.

1. Using excessive hand washing to relieve anxiety is a behavioral symptom exhibited by clients diagnosed with obsessivecompulsive disorder (OCD). TEST-TAKING HINT: To answer this question correctly, the test taker must be able to differentiate various classes of symptoms exhibited by clients diagnosed with OCD. The keyword "behavioral" determines the correct answer to this question.

The nurse on the in-patient psychiatric unit should include which of the following interventions when working with a newly admitted client diagnosed with obsessivecompulsive disorder? Select all that apply. 1. Assess previously used coping mechanisms and their effects on anxiety. 2. Allow time for the client to complete compulsions. 3. With the client's input, set limits on ritualistic behaviors. 4. Present the reality of the impact the compulsions have on the client's life. 5. Discuss client feelings surrounding the obsessions and compulsions.

1. When a client is newly admitted, it is important for the nurse to assess past coping mechanisms and their effects on anxiety. Assessment is the first step in the nursing process, and this information needs to be gathered to intervene effectively. 2. Allowing time for the client to complete compulsions is important for a client who is newly admitted. If compulsions are limited, anxiety levels increase. If the client had been hospitalized for a while, then, with the client's input, limits would be set on the compulsive behaviors. 5. It is important for the nurse to allow the client to express his or her feelings about the obsessions and compulsions. This assessment of feelings should begin at admission. TEST-TAKING HINT: It is important for the test taker to note the words "newly admitted" in the question. The nursing interventions implemented vary and are based on length of stay on the unit, along with client's insight into his or her disorder. For clients with obsessive-compulsive disorder, it is important to understand that the compulsions are used to decrease anxiety. If the compulsions are limited, anxiety increases. Also, the test taker must remember that during treatment it is imperative that the treatment team includes the client in decisions related to any limitation of compulsive behaviors.

A client who is pacing and wringing his hands states, "I just need to walk" when questioned by the nurse about what he is feeling. Which of the following responses by the nurse is most therapeutic? 1. "You need to sit down and relax." 2. "Are you feeling anxious?" 3. "Is something bothering you?" 4. "You must be experiencing a problem now."

2. Asking, "Are you feeling anxious?" helps the client to specifically label the feeling as anxiety so that he can begin to understand and manage it. Some clients need assistance with identifying what they are feeling so they can recognize what is happening to them. Stating, "You need to sit down and relax," is not appropriate because the client needs to continue his pacing to feel better. Asking if something is bothering the client or saying that he must be experiencing a problem is vague and does not help the client identify his feelings as anxiety.

A client diagnosed with Obsessive-Compulsive Disorder has been taking sertraline (Zoloft) but would like to have more energy every day. At his monthly checkup, he reports that his massage therapist recommended he take St. John's Wort to help his depression. The nurse should tell the client: 1. "St. John's Wort is a harmless herb that might be helpful in this instance." 2. "Combining St. John's Wort with the Zoloft can cause a serious reaction called Serotonin Syndrome." 3. "If you take St. John's, we'll have to decrease the dose of your Zoloft." 4. "St. John's Wort isn't very effective for depression, but we can increase your Zoloft dose."

2. The effectiveness of St. John's Wort with depression is unconfirmed. The critical issue is that the combination of St. John's Wort and Zoloft (an SSRI antidepressant) can produce Serotonin Syndrome which can be fatal. The client should not take the St. John's Wort while taking Zoloft.

After months of counseling, a client abused by her husband tells the nurse that she has decided to stop treatment. There has been no abuse during this time, and she feels better able to cope with the needs of her husband and children. In discussing this decision with the client, the nurse should: 1. Tell the client that this is a bad decision that she will regret in the future. 2. Find out more about the client's rationale for her decision to stop treatment. 3. Warn the client that abuse commonly stops when one partner is in treatment, only to begin again later. 4. Remind the client of her duty to protect her children by continuing treatment.

2. The nurse needs more information about the client's decision before deciding what intervention is most appropriate. Judgmental responses could make it difficult for the client to return for treatment should she want to do so. Telling the client that this is a bad decision that she will regret is inappropriate because the nurse is making an assumption. Warning the client that abuse commonly stops when one partner is involved in treatment may be true for some clients. However, until the nurse determines the basis for the client's decision, this type of response is an assumption and therefore inappropriate. Reminding the client about her duty to protect the children would be appropriate if the client had talked about episodes of current abuse by her partner and the fear that her children might be hurt by him.

"After months of coaxing by her husband, a client comes to the mental health clinic. She reports that she suffers from an overwhelming fear of leaving her house. This overwhelming fear has caused the client to lose her job and is beginning to take a toll on her marriage. The physician diagnoses the client with agoraphobia. Which treatment options are effective in treating this disorder?

2. Desensitization 3. Alprazolam (Xanax) therapy 4. Paroxetine (Paxil) therapy

A client diagnosed with obsessive-compulsive disorder has been hospitalized for the last 4 days. Which intervention would be a priority at this time? 1. Notify the client of the expected limitations on compulsive behaviors. 2. Reinforce the use of learned relaxation techniques. 3. Allow the client the time needed to complete the compulsive behaviors. 4. Say "stop" to the client as a thought-stopping technique.

2. It is important for the client to learn techniques to reduce overall levels of anxiety to decrease the need for compulsive behaviors. The teaching of these techniques should begin by day 4. TEST-TAKING HINT: To answer this question correctly, the test taker must understand that nursing interventions should be based on timeframes appropriate to the expressed symptoms and severity of the client's disorder. The length of hospitalization also must be considered in planning these interventions. The average stay on an in-patient psychiatric unit is 5 to 7 days.

During an intake assessment, a client diagnosed with generalized anxiety disorder rates mood at 3/10, rates anxiety at 8/10, and states, "I'm thinking about suicide." Which nursing intervention takes priority? 1. Teach the client relaxation techniques. 2. Ask the client, "Do you have a plan to commit suicide?" 3. Call the physician to obtain a PRN order for an anxiolytic medication. 4. Encourage the client to participate in group activities.

2. It is important for the nurse to ask the client about a potential plan for suicide to intervene in a timely manner. Clients who have developed suicide plans are at higher risk than clients who may have vague suicidal thoughts. TEST-TAKING HINT: To answer this question correctly, the test taker must understand the importance of assessing the plan for suicide. Interventions would differ depending on the client's plan. The intervention for a plan to use a gun at home would differ from an intervention for a plan to hang oneself during hospitalization.

Which teaching need is important when a client is newly prescribed buspirone (BuSpar) 5 mg tid? 1. Encourage the client to avoid drinking alcohol while taking this medication because of the additive central nervous system depressant effects. 2. Encourage the client to take the medication continually as prescribed because onset of action is delayed 2 to 3 weeks. 3. Encourage the client to monitor for signs and symptoms of anxiety to determine need for additional buspirone (BuSpar) PRN. 4. Encourage the client to be compliant with monthly lab tests to monitor for medication toxicity.

2. It is important to teach the client that the onset of action for buspirone (BuSpar) is 2 to 3 weeks. Often the nurse may see a benzodiazepine, such as clonazepam, prescribed because of its quick onset of effect, until the buspirone begins working. TEST-TAKING HINT: To answer this question correctly, the test taker must understand that buspirone (BuSpar) has a delayed onset of action, which can affect medication compliance. If the effects of the medication are delayed, the client is likely to stop taking the medication. Teaching about delayed onset is an important nursing intervention.

While interviewing a 3-year-old girl who has been sexually abused about the event, which approach would be most effective? 1. Describe what happened during the abusive act. 2. Draw a picture and explain what it means. 3. "Play out" the event using anatomically correct dolls. 4. Name the perpetrator.

3. A 3-year-old child has limited verbal skills and should not be asked to describe an event, explain a picture, or respond verbally or nonverbally to questions. More appropriately, the child can act out an event using dolls. The child is likely to be too fearful to name the perpetrator or will not be able to do so.

During the third session with the nurse, a client who is being abused states, "I don't know what to do anymore. He doesn't want me to go anywhere while he's at work, not even to visit my friends." Which nursing diagnosis should the nurse formulate regarding this information? 1. Risk for other-directed violence related to an abusive husband, as evidenced by the victim's statement of being battered. 2. Situational low self-esteem related to victimization, as evidenced by not being able to leave the house. 3. Powerlessness related to control by husband, as evidenced by the inability to make decisions. 4. Ineffective coping related to victimization, as evidenced by crying.

3. Based on the client's statements, such as "I don't know what to do anymore," the data here best support the nursing diagnosis of Powerlessness related to control by husband, as evidenced by inability to make decisions. A nursing diagnosis of Risk for other-directed violence would be appropriate if the client had talked about being beaten up the previous night. A nursing diagnosis of Situational low self-esteem would be appropriate if the client verbalized feelings of embarrassment in leaving the house and worthlessness. A nursing diagnosis of Ineffective coping would be appropriate if the client was crying or talked about crying herself to sleep at night.

Adolescents and adults who were sexually abused as children commonly mutilate themselves. The nurse interprets this behavior as: 1. The need to make themselves less sexually attractive. 2. An alternative to bingeing and purging. 3. Use of physical pain to avoid dealing with emotional pain. 4. An alternative to getting high on drugs.

3. Dealing with the physical pain associated with mutilation is viewed as easier than dealing with the intense anger and emotional pain. The client fears an aggressive outburst when anger and emotional pain increase. Self-mutilation seems easier and safer. Additionally, self-mutilation may occur if the client feels unreal or numb or is dissociating. Here, the mutilation proves to the client that he or she is alive and capable of feeling. The client may want to be less sexually attractive, but this aspect usually is not related to self-mutilation. Bingeing and purging is commonly done in addition to, not instead of, self-mutilation. Although a few clients report an occasional high with self-mutilation, usually the experience is just relief from anger and rage.

In working with a rape victim, which of the following is most important? 1. Continuing to encourage the client to report the rape to the legal authorities. 2. Recommending that the client resume sexual relations with her partner as soon as possible. 3. Periodically reminding the client that she did not deserve and did not cause the rape. 4. Telling the client that the rapist will eventually be caught, put on trial, and jailed.

3. Guilt and self-blame are common feelings that need to be addressed directly and frequently. The client needs to be reminded periodically that she did not deserve and did not cause the rape. Continually encouraging the client to report the rape pressures the client and is not helpful. In most cases, resuming sexual relations is a difficult process that is not likely to occur quickly. It is not necessarily true that the rapist will be caught, tried, and jailed. Most rapists are not caught or convicted.

A third-grade child is referred to the mental health clinic by the school nurse because he is fearful, anxious, and socially isolated. After meeting with the client, the nurse talks with his mother, who says, "It's that school nurse again. She's done nothing but try to make trouble for our family since my son started school. And now you're in on it." The nurse should respond by saying: 1. "The school nurse is concerned about your son and is only doing her job." 2. "We see a number of children who go to your son's school. He isn't the only one." 3. "You sound pretty angry with the school nurse. Tell me what has happened." 4. "Let me tell you why your son was referred, and then you can tell me about your concerns."

3. The mother's feelings are the priority here. Addressing the mother's feelings and asking for her view of the situation is most important in building a relationship with the family. Ignoring the mother's feelings will hinder the relationship. Defending the school nurse and the school puts the client's mother on the defensive and stifles communication.

An adult client diagnosed with anxiety disorder becomes anxious when she touches fruits and vegetables. What should the nurse do? 1. Instruct the woman to avoid touching these foods. 2. Ask the woman why she becomes anxious in these situations. 3. Assist the woman to make a plan for her family to do the food shopping and preparation. 4. Teach the woman to use cognitive behavioral approaches to manage her anxiety.

4. Cognitive behavioral therapy is effective in treating anxiety disorders. The nurse can assist the client in identifying the onset of the fears that cause the anxiety and develop strategies to modify the behavior associated with the fears. Avoiding touching foods, asking about reasons for the anxiety, and providing ways to work around touching the foods do not deal with the anxiety and are not interventions that will help this client.

A client with acute stress disorder states to the nurse, "I keep having horrible nightmares about the car accident that killed my daughter. I shouldn't have taken her with me to the store." Which of the following responses by the nurse is most therapeutic? 1. "Don't keep torturing yourself with such horrible thoughts." 2. "Stop blaming yourself. It's only hurting you." 3. "Let's talk about something that is a bit more pleasant." 4. "The accident just happened and could not have been predicted."

4. Saying, "The accident just happened and could not have been predicted," provides the client with an objective perception of the event instead of the client's perceived role. This type of statement reflects active listening and helps to reduce feelings of blame and guilt. Saying, "Don't keep torturing yourself," or "Stop blaming yourself," is inappropriate because it tells the client what to do, subsequently delaying the therapeutic process. The statement, "Let's talk about something that is a bit more pleasant," ignores the client's feelings and changes the subject. The client needs to verbalize feelings and decrease feelings of isolation.

During an assessment, a client diagnosed with generalized anxiety disorder rates anxiety as 9/10 and states, "I have thought about suicide because nothing ever seems to work out for me." Based on this information, which nursing diagnosis takes priority? 1. Hopelessness R /T anxiety AEB client's stating, "Nothing ever seems to work out." 2. Ineffective coping R /T rating anxiety as 9/10 AEB thoughts of suicide. 3. Anxiety R /T thoughts about work AEB rates anxiety 9/10. 4. Risk for suicide R /T expressing thoughts of suicide.

4. Because the client is expressing suicidal ideations, the nursing diagnosis of risk for suicide takes priority at this time. Client safety is prioritized over all other client problems. TEST-TAKING HINT: When looking for a priority nursing diagnosis, the test taker always must prioritize client safety. Even if other problems exist, client safety must be ensured.

A client diagnosed with posttraumatic stress disorder states to the nurse, "All those wonderful people died, and yet I was allowed to live." Which is the client experiencing? 1. Denial. 2. Social isolation. 3. Anger. 4. Survivor's guilt.

4. The client in the question is experiencing survivor's guilt. Survivor's guilt is a common situation that occurs when an individual experiences a traumatic event in which others died and the individual survived. TEST-TAKING HINT: To answer this question correctly, the test taker needs to understand common phenomena experienced by individuals diagnosed with posttraumatic stress disorder and relate this understanding to the client statement presented in the question.

A high school senior is diagnosed with anorexia nervosa and is hospitalized for severe malnutrition. The treatment team is planning to use behavior modification. What rationale should a nurse identify as the reasoning behind this therapy choice? A. This therapy will increase the client's motivation to gain weight. B. This therapy will reward the client for perfectionist achievements. C. This therapy will provide the client with control over behavioral choices. D. This therapy will protect the client from parental overindulgence.

A high school senior is diagnosed with anorexia nervosa and is hospitalized for severe malnutrition. The treatment team is planning to use behavior modification. What rationale should a nurse identify as the reasoning behind this therapy choice? A. This therapy will increase the client's motivation to gain weight. B. This therapy will reward the client for perfectionist achievements. C. This therapy will provide the client with control over behavioral choices. D. This therapy will protect the client from parental overindulgence.

PTSD Define

A severe anxiety disorder that occurs following exposure to a major traumatic event, which results in repeated flashbacks, nightmares, or emotionally crippling fear responses. PTSD changes the body's response to stress. It affects the stress hormones and chemicals that carry neurotransmitters. - Cause of PTSD is unknown. - Psychological, genetic, physical, and social factors are involved. - History or recent trauma may increase risk: assault, domestic abuse, prison stay, rape, terrorism, and war.

A client is admitted to the psychiatric unit with a diagnosis of major depression. The client is unable to concentrate, has no appetite, and is experiencing insomnia. Which should be included in this client's plan of care? A. A simple, structured daily schedule with limited choices of activities B. A daily schedule filled with activities to promote socialization C. A flexible schedule that allows the client opportunities for decision making D. A schedule that includes mandatory activities to decrease social isolation

ANS: A A client diagnosed with depression has difficulty concentrating and may be overwhelmed by activity overload or the expectation of independent decision making. A simple, structured daily schedule with limited choices of activities is more appropriate.

Which nursing statement reflects a common characteristic of a client diagnosed with paranoid personality disorder? A. "This client consistently criticizes care and has difficulty getting along with others." B. "This client is shy and fades into the background." C. "This client expects special treatment and setting limits will be necessary." D. "This client is expressive during group and is very pleased with self."

ANS: A A client diagnosed with paranoid personality disorder has a pervasive distrust and suspiciousness of others. Anticipating humiliation and betrayal, the paranoid individual characteristically learns to attack first.

Which characteristics should a nurse recognize as being exhibited by individuals diagnosed with any personality disorders? A. These clients accept and are comfortable with their altered behaviors. B. These clients understand that their altered behaviors result from anxiety. C. These clients seek treatment to avoid interpersonal discomfort. D. These clients avoid relationships due to past negative experiences.

ANS: A Clients who are diagnosed with personality disorders accept and are comfortable with their altered behaviors. Personalities that develop in a disordered pattern remain somewhat unstable and unpredictable throughout the lifetime.

A 15-year-old who is angry about not being chosen as the basketball team's captain, spray paints obscene words on the newly chosen captain's car. What information would cause a school nurse to consider a diagnosis of intermittent explosive disorder? A. The destruction of property is grossly out of proportion to the precipitating factor. B. The destruction of property is not a pattern of failure to resist aggressive impulses. C. The teenager has a diagnosis of conduct disorder. D. The teenager has previously been diagnosed with Tourette's syndrome.

ANS: A The DSM-IV-TR criteria for the diagnosis of intermittent explosive disorder state that several discrete episodes of destruction of property must occur, and the aggressive episode can not be better accounted for by another mental disorder such as conduct disorder or Tourette's syndrome. The degree of aggressiveness must be grossly out of proportion to the precipitating factor.

Family members of a client ask a nurse to explain the difference between schizoid and avoidant personality disorders. Which is the appropriate nursing reply? A. "Clients diagnosed with avoidant personality disorder desire intimacy but fear it, and clients diagnosed with schizoid personality disorder prefer to be alone." B. "Clients diagnosed with schizoid personality disorder exhibit odd, bizarre, and eccentric behavior, while clients diagnosed with avoidant personality disorder do not." C. "Clients diagnosed with avoidant personality disorder are eccentric, and clients diagnosed with schizoid personality disorder are dull and vacant." D. "Clients diagnosed with schizoid personality disorder have a history of psychotic thought processes, while clients diagnosed with avoidant personality disorder remain based in reality."

ANS: A The nurse should educate the family that clients diagnosed with avoidant personality disorder desire intimacy but fear it, while clients diagnosed with schizoid personality disorder prefer to be alone. Avoidant personality disorder is characterized by an extreme sensitivity to rejection which leads to social isolation. Schizoid personality disorder is characterized by a profound deficit in the ability to form personal relationships.

A nursing instructor is teaching students about the differences between the symptoms of anorexia nervosa and the symptoms of bulimia nervosa. Which student statement indicates that learning has occurred? A. "Clients diagnosed with anorexia nervosa experience extreme nutritional deficits, whereas clients diagnosed with bulimia nervosa do not." B. "Clients diagnosed with bulimia nervosa experience amenorrhea, whereas clients diagnosed with anorexia nervosa do not." C. "Clients diagnosed with bulimia nervosa experience hypotension, edema, and lanugo, whereas clients diagnosed with anorexia nervosa do not." D. "Clients diagnosed with anorexia nervosa have eroded tooth enamel, whereas clients diagnosed with bulimia nervosa do not."

ANS: A The nursing student statement that clients diagnosed with anorexia nervosa experience nutritional deficits, whereas clients diagnosed with bulimia nervosa do not, indicates that learning has occurred. Anorexia is characterized by low caloric and nutritional intake. Bulimia is characterized by episodic, rapid indigestion of large quantities of food followed by purging.

Which nursing diagnosis should be prioritized when providing care to a client diagnosed with paranoid personality disorder? A. Risk for violence: directed toward others R/T suspicious thoughts B. Risk for suicide R/T altered thought C. Altered sensory perception R/T increased levels of anxiety D. Social isolation R/T inability to relate to others

ANS: A The priority nursing diagnosis for a client diagnosed with paranoid personality disorder should be risk for violence: directed toward others R/T suspicious thoughts. Clients diagnosed with paranoid personality disorder have a pervasive distrust and suspiciousness of others that may result in hostile actions to protect self. They are often tense and irritable, which increases the likelihood of violent behavior.

A nurse should identify topiramate (Topamax) as the drug of choice for which of the following conditions? (Select all that apply.) A. Binge eating with obesity B. Bingeing and purging with a diagnosis of bulimia nervosa C. Weight loss with a diagnosis of anorexia nervosa D. Amenorrhea with a diagnosis of anorexia nervosa E. Emaciation with a diagnosis of bulimia nervosa

ANS: A, B The nurse should identify that topiramate (Topamax) is the drug of choice when treating binge eating with obesity and bingeing and purging with a diagnosis of bulimia nervosa. Topiramate (Topamax) is a novel anticonvulsant used in the long-term treatment of binge-eating disorder with obesity. The use of Topamax results in a significant decline in mean weekly binge frequency and significant reduction in body weight. With the use of this medication, episodes of bingeing and purging were decreased in clients diagnosed with bulimia nervosa.

Which statements represent positive outcomes for clients diagnosed with narcissistic personality disorder? (Select all that apply.) A. The client will relate one empathetic statement toward another client in group by day 2. B. The client will identify one personal limitation by day 1. C. The client will acknowledge one strength that another client possesses by day 2. D. The client will list four personal strengths by day 3. E. The client will list two lifetime achievements by discharge.

ANS: A, B, C The nurse should determine that appropriate outcomes for a client diagnosed with narcissistic personality disorder include relating empathetic statements to other clients, identifying one personal limitation, and acknowledging one strength in another client. An exaggerated sense of self-worth, a lack of empathy, and exploitation of others are characteristics of narcissistic personality disorder.

Which concepts should a nurse identify as being included in the DSM-IV-TR definition of personality? (Select all that apply.) A. Personality is an enduring pattern of perceiving. B. Personality is influenced by relationships between the environment and self. C. Personality is developed in sporadic stages that vary from person to person. D. Personality is influenced by a wide range of social and personal contexts. E. Personality is inborn and cannot be influenced by developmental progression.

ANS: A, B, D The nurse should identify that the following concepts are included in the DSM-IV-TR definition of personality: Personality is an enduring pattern of perceiving, a wide range of social and personal contexts influences it, and it is inborn. Personality disorders are coded on Axis II of the DSM-IV-TR multiaxial diagnosis and include disorders organized into three clusters: odd and eccentric disorders (cluster A); dramatic, emotional, or erratic disorders (cluster B); and anxious or fearful disorders (cluster C).

When planning care for women in abusive relationships, which of the following information is important for the nurse to consider? (Select all that apply.) A. It often takes several attempts before a woman leaves an abusive situation. B. Substance abuse is a common factor in abusive relationships. C. Until children reach school age, they are usually not affected by parental discord. D. Women in abusive relationships usually feel isolated and unsupported. E. Economic factors rarely play a role in the decision to stay in abusive relationships.

ANS: A, B, D When planning care for women who have been victims of domestic abuse, the nurse should be aware that it often takes several attempts before a woman leaves an abusive situation, that substance abuse is a common factor in abusive relationships, and that women in abusive relationships usually feel isolated and unsupported. Children can be affected by domestic violence from infancy, and economic factors often play a role in the victim's decision to stay.

Which of the following client statements would indicate that teaching about benzodiazepines has been successful? (Select all that apply.) A. "I can't drink alcohol when taking lorazepam (Ativan)." B. "If I abruptly stop taking buspirone (BuSpar), I may have a seizure." C. "Valium can make me drowsy, so I shouldn't drive for awhile." D. "My new diet cannot include aged cheese or pickled herring." E. "When the fluoxetine (Prozac) begins working, I can stop the alprazolam (Xanax)."

ANS: A, C When a nurse teaches about medications the nurse is using a cognitive approach. A core concept of cognitive theory relates to the mental process of thinking and reasoning.

A nurse is caring for a group of clients within the DSM-IV-TR cluster B category of personality disorders. Which factors should the nurse consider when planning client care? (Select all that apply.) A. These clients have personality traits that are deeply ingrained and difficult to modify. B. These clients need medications to treat the underlying physiological pathology. C. These clients use manipulation, making the implementation of treatment problematic. D. These clients have poor impulse control that hinders compliance with a plan of care. E. These clients commonly have secondary diagnoses of substance abuse and depression.

ANS: A, C, D, E The nurse should consider that individuals diagnosed with cluster B-type personality disorders have deeply ingrained personality traits, use manipulation, have poor impulse control, and often have secondary diagnoses of substance abuse and/or depression. This cluster includes antisocial, borderline, histrionic, and narcissistic personality disorders.

A nursing home resident has a diagnosis of dysthymic disorder. When planning care for this client, which of the following symptoms should a nurse expect the client to exhibit? (Select all that apply.) A. Sad mood on most days B. Mood rating of 2/10 for the past 6 months C. Labile mood D. Sad mood for the past 3 years after spouse's death E. Pressured speech when communicating

ANS: A, D The nurse should anticipate that a client with a diagnosis of dysthymic disorder would experience a sad mood on most days for more than 2 years. The essential feature of dysthymia is a chronically depressed mood which can have an early or late onset.

In evaluating nursing interventions, which of the following types of questions would a nurse use to gather information from a client diagnosed with an impulse control disorder? (Select all that apply.) A. Can the client demonstrate the ability to delay gratification? B. Does the client demonstrate evidence of progression along the grief response? C. Can the client accomplish activities of daily living independently? D. Does the client verbalize symptoms of tension preceding unacceptable behavior? E. Does the client verbalize the unacceptability of maladaptive behaviors?

ANS: A, D, E A client diagnosed with an impulse control disorder should not have difficulty accomplishing activities of daily living or progressing through the grief process. These types of questions would be appropriate for clients diagnosed with adjustment disorders, not impulse control disorders.

Which nursing intervention is appropriate when caring for clients diagnosed with either anorexia nervosa or bulimia nervosa? A. Provide privacy during meals. B. Remain with the client for at least 1 hour after the meal. C. Encourage the client to keep a journal to document types of food consumed. D. Restrict client privileges when provided food is not completely consumed.

ANS: B A nurse should remain with clients diagnosed with either anorexia nervosa or bulimia nervosa for at least 1 hour after meals. This allows the nurse to monitor for food discarding (anorexia nervosa) and/or self-induced vomiting (bulimia nervosa).

A client is diagnosed with major depressive disorder. Which nursing diagnosis should a nurse assign to this client to address a behavioral symptom of this disorder? A. Altered communication R/T feelings of worthlessness AEB anhedonia B. Social isolation R/T poor self-esteem AEB secluding self in room C. Altered thought processes R/T hopelessness AEB persecutory delusions D. Altered nutrition: less than body requirements R/T high anxiety AEB anorexia

ANS: B A nursing diagnosis of social isolation R/T poor self-esteem AEB secluding self in room addresses a behavioral symptom of major depressive disorder. Other behavioral symptoms include psychomotor retardation, virtually nonexistent communication, maintaining a fetal position, and no personal hygiene and/or grooming.

A client has been extremely nervous ever since a person died as a result of the client's drunk driving. When assessing for the diagnosis of adjustment disorder, within what timeframe should the nurse expect the client to exhibit these symptoms? A. To meet the DSM-IV criteria for adjustment disorder, the client should exhibit symptoms within 1 year of the accident. B. To meet the DSM-IV criteria for adjustment disorder, the client should exhibit symptoms within 3 months of the accident. C. To meet the DSM-IV criteria for adjustment disorder, the client should exhibit symptoms within 6 months of the accident. D. To meet the DSM-IV criteria for adjustment disorder, the client should exhibit symptoms within 9 months of the accident.

ANS: B According to the DSM-IV diagnostic criteria for adjustment disorders, the development of emotional or behavioral symptoms in response to an identifiable stressor occurs within 3 months of the onset of the stressor.

A client who is 5 foot 6 inches tall and weighs 98 pounds is admitted with a medical diagnosis of anorexia nervosa. Which nursing diagnosis would take priority at this time? A. Ineffective coping R/T food obsession B. Altered nutrition: less than body requirements R/T inadequate food intake C. Risk for injury R/T suicidal tendencies D. Altered body image R/T perceived obesity

ANS: B Based on Maslow's hierarchy, the priority nursing diagnosis for this client must address physical needs prior to emotional considerations. This client must be immediately physically stabilized due to the life-threatening nature of his or her nutritional status.

A client who has been taking buspirone (BuSpar) as prescribed for 2 days is close to discharge. Which statement indicates to the nurse that the client has an understanding of important discharge teaching? A. "I cannot drink any alcohol with this medication." B. "It is going to take 2 to 3 weeks in order for me to begin to feel better." C. "This drug causes physical dependence and I need to strictly follow doctor's orders." D. "I can't take this medication with food. It needs to be taken on an empty stomach."

ANS: B Buspar takes at least 2 to 3 weeks to be effective in controlling symptoms of depression. This is important to teach clients in order to prevent potential noncompliance due to the perception that the medication is ineffective.

While improving, a client demands to have a phone installed in the intensive care unit (ICU) room. When a nurse states, "This is not allowed. It is a unit rule." The client angrily demands to see the doctor. Which approach should the nurse use in this situation? A. Provide an explanation for the necessity of the unit rule. B. Assist the client to discuss anger and frustrations. C. Call the physician and relay the request. D. Arrange for a phone to be installed in the client's unit room.

ANS: B Clients who demand special privileges may be diagnosed with narcissistic personality disorder. The best approach in this situation is for the nurse to identify the function that anger, frustration, and rage serve for the client. The verbalization of feelings may help the client to gain insight into his or her behavior.

After a spouse dies, a client is diagnosed with adjustment disorder with depressed mood. Client symptoms include chronic migraines, feelings of hopelessness, social isolation, and self-care deficit. Which outcome would be most appropriate to direct the focus of this client's care? A. The client will not cope with stress by impulsive behaviors by discharge. B. The client will accomplish activities of daily living independently by discharge. C. The client will be able to cope effectively by delaying gratification by discharge. D. The client will verbalize a positive body image by discharge.

ANS: B Impulsive behaviors and the inability to delay gratification are symptoms of impulse control, not adjustment disorders. There is no evidence presented that the client has a body image distortion. Setting an outcome of independent self-care will direct nursing interventions toward encouraging the client to meet self-care needs.

The nurse should recognize which factors that distinguish personality disorders from psychosis? A. Functioning is more limited in personality disorders than in psychosis. B. Major disturbances of thought are absent in personality disorders. C. Personality disordered clients require hospitalization more frequently. D. Personality disorders do not affect family relationships as much as psychosis.

ANS: B Major disturbances of thought are absent in personality disorders and are a classic symptom of psychosis.

A pessimistic client expresses low self-worth, has much difficulty making decisions, avoids positions of responsibility, and has a behavioral pattern of "suffering" in silence. Which underlying cause of this client's personality disorder should a nurse recognize? A. "Nurturance was provided from many sources, and independent behaviors were encouraged." B. "Nurturance was provided exclusively from one source, and independent behaviors were discouraged." C. "Nurturance was provided exclusively from one source, and independent behaviors were encouraged." D. "Nurturance was provided from many sources, and independent behaviors were discouraged."

ANS: B Nurturance provided from one source and discouragement of independent behaviors can attribute to the etiology of dependent personality disorder. Dependent behaviors may be rewarded by a parent who is overprotective and discourages autonomy.

During an assessment interview, a client diagnosed with antisocial personality disorder spits, curses, and refuses to answer questions. Which is the most appropriate nursing statement to address this behavior? A. "You are very disrespectful. You need to learn to control yourself." B. "I understand that you are angry, but this behavior will not be tolerated." C. "What behaviors could you modify to improve this situation?" D. "What anti-personality-disorder medications have helped you in the past?"

ANS: B The appropriate nursing statement is to reflect the client's feeling while setting firm limits on behavior. Clients diagnosed with antisocial personality disorder have a low tolerance for frustration, see themselves as victims, and use projection as a primary ego defense mechanism. Antidepressants and anxiolytics are used for symptom relief; however, there are no specific medications targeted for the diagnosis of a personality disorder.

A nursing instructor is teaching students about clients diagnosed with histrionic personality disorder and the quality of their relationships. Which student statement indicates that learning has occurred? A. "Their dramatic style tends to make their interpersonal relationships quite interesting and fulfilling." B. "Their interpersonal relationships tend to be shallow and fleeting, serving their dependency needs." C. "They tend to develop few relationships because they are strongly independent but generally maintain deep affection." D. "They pay particular attention to details which can frustrate the development of relationships."

ANS: B The instructor should evaluate that learning has occurred when the student describes clients diagnosed with histrionic personality disorder as having shallow, fleeting interpersonal relationships that serve their dependency needs. Histrionic personality disorder is characterized by colorful, dramatic, and extroverted behavior. These individuals also have difficulty maintaining long-lasting relationships.

A client is brought to an emergency department after being violently raped. Which nursing action is appropriate? A. Discourage the client from discussing the event as this may lead to further emotional trauma. B. Remain nonjudgmental and actively listen to the client's description of the event. C. Meet the client's self-care needs by assisting with showering and perineal care. D. Provide cues, based on police information, to encourage further description of the event.

ANS: B The most appropriate nursing action is to remain nonjudgmental and actively listen to the client's description of the event. It is important to also communicate to the victim that he or she is safe and that it is not his or her fault. Nonjudgmental listening provides an avenue for client catharsis needed in order to begin the process of healing.

From a behavioral perspective, which nursing intervention is most appropriate when caring for a client diagnosed with borderline personality disorder? A. Seclude the client when inappropriate behaviors are exhibited. B. Contract with the client to reinforce positive behaviors with unit privileges. C. Teach the purpose of antianxiety medications to improve medication compliance. D. Encourage the client to journal feelings to improve awareness of abandonment issues.

ANS: B The most appropriate nursing intervention from a behavioral perspective is to contract with the client to reinforce positive behaviors with unit privileges. Behavioral strategies offer reinforcement for positive change.

An individual experiences sadness and melancholia in September continuing through November. Which of the following factors should a nurse identify as most likely to contribute to the etiology of these symptoms? (Select all that apply.) A. Gender differences in social opportunities that occur with age B. Drastic temperature and barometric pressure changes C. Increased levels of melatonin D. Variations in serotonergic functioning E. Inaccessibility of resources for dealing with life stressors

ANS: B, C, D The nurse should identify drastic temperature and barometric pressure changes, increased levels of melatonin, and/or variations in serotonergic functioning as contributing to the etiology of the client's symptoms. A number of studies have examined seasonal patterns associated with mood disorders and have revealed two prevalent periods of seasonal involvement: spring (March, April, May) and fall (September, October, November).

Which adult client should a nurse identify as exhibiting the characteristics of a dependent personality disorder? A. A physically healthy client who is dependent on meeting social needs by contact with 15 cats B. A physically healthy client who has a history of depending on intense relationships to meet basic needs C. A physically healthy client who lives with parents and relies on public transportation D. A physically healthy client who is serious, inflexible, perfectionistic, and depends on rules to provide security

ANS: C A physically healthy adult client who lives with parents and relies on public transportation exhibits signs of dependent personality disorder. Dependent personality disorder is characterized by a pervasive and excessive need to be taken care of that leads to submissive and clinging behavior.

When a community health nurse arrives at the home of a client diagnosed with bulimia nervosa, the nurse finds the client on the floor unconscious. The client has a history of using laxatives for purging. To what would the nurse attribute this client's symptoms? A. Increased creatinine and blood urea nitrogen (BUN) levels B. Abnormal electroencephalogram (EEG) C. Metabolic acidosis D. Metabolic alkalosis

ANS: C Excessive vomiting and laxative or diuretic abuse may lead to problems with dehydration and electrolyte imbalance. The nurse should attribute this client's fainting to the loss of alkaline stool due to laxative abuse which would lead to a relative metabolic acidotic condition.

During her aunt's wake, before a mother can stop her 4-year-old child, the child runs up to the casket. An appointment is made with a nurse practitioner when the child starts twisting and pulling out hair. Which nursing diagnosis should the nurse practitioner assign to this child? A. Complicated grieving B. Altered family processes C. Ineffective coping D. Body image disturbance

ANS: C Ineffective coping is defined as an inability to form a valid appraisal of the stressors, inadequate choices of practiced responses, and/or the inability to use available resources. This child is coping with the anxiety generated by viewing her deceased aunt by pulling out hair. If this behavior continues, a diagnosis of the impulse control disorder, trichotillomania, may be assigned.

When questioned about bruises, a woman states, "It was an accident. My husband just had a bad day at work. He's being so gentle now and even brought me flowers. He's going to get a new job, so it won't happen again." This client is in which phase of the cycle of battering? A. Phase I: The tension-building phase B. Phase II: The acute battering incident phase C. Phase III: The honeymoon phase D. Phase IV: The resolution and reorganization phase

ANS: C The client is in the honeymoon phase of the cycle of battering. In this phase, the batterer becomes extremely loving, kind, and contrite. Promises are often made that the abuse will not happen again.

A client's altered body image is evidenced by claims of "feeling fat" even though the client is emaciated. Which is the appropriate outcome criterion for this client's problem? A. The client will consume adequate calories to sustain normal weight. B. The client will cease strenuous exercise programs. C. The client will perceive an ideal body weight and shape as normal. D. The client will not express a preoccupation with food.

ANS: C The nurse should identify that the appropriate outcome for this client is to perceive an ideal body weight and shape as normal. Additional goals include accepting self based on self-attributes instead of appearance and to realize that perfection is unrealistic.

A client diagnosed with bulimia nervosa has been attending a mental health clinic for several months. Which factor should a nurse identify as an appropriate indicator of a positive client behavioral change? A. The client gains 2 pounds in 1 week. B. The client focuses conversations on nutritious food. C. The client demonstrates healthy coping mechanisms that decrease anxiety. D. The client verbalizes an understanding of the etiology of the disorder.

ANS: C The nurse should identify that when a client uses healthy coping mechanisms that decrease anxiety, positive behavioral change is demonstrated. Stress and anxiety can increase bingeing which is followed by inappropriate compensatory behaviors.

A client who has been raped is crying, pacing, and cursing her attacker in an emergency department. Which behavioral defense should a nurse recognize? A. Controlled response pattern B. Compounded rape reaction C. Expressed response pattern D. Silent rape reaction

ANS: C The nurse should recognize that this client is exhibiting an expressed response pattern. In the expressed response pattern, feelings of fear, anger, and anxiety are expressed through crying, sobbing, smiling, restlessness, and tension. In the controlled response pattern, the client's feelings are masked or hidden, and a calm, composed, or subdued affect is seen.

Which client symptoms should lead a nurse to suspect a diagnosis of obsessive-compulsive personality disorder? A. The client experiences unwanted, intrusive, and persistent thoughts. B. The client experiences unwanted, repetitive behavior patterns. C. The client experiences inflexibility and lack of spontaneity when dealing with others. D. The client experiences obsessive thoughts that are externally imposed.

ANS: C The nurse should suspect a diagnosis of obsessive-compulsive personality disorder when a client experiences inflexibility and lack of spontaneity. Individuals diagnosed with this disorder are very serious, formal, and have difficulty expressing emotions. They are perfectionistic and preoccupied with rules.

When planning care for clients diagnosed with personality disorders, what should be the anticipated treatment outcome? A. To stabilize pathology with the correct combination of medications B. To change the characteristics of the dysfunctional personality C. To reduce inflexibility of personality traits that interfere with functioning and relationships D. To decrease the prevalence of neurotransmitters at receptor sites

ANS: C The outcome of treatment for clients diagnosed with personality disorders should be to reduce inflexibility of personality traits that interfere with functioning and relationships. Personality disorders are often difficult and, in some cases, seem impossible to treat.

A nurse is seeing a client in an outpatient clinic for treatment of anorexia nervosa. Which is the most appropriate, correctly written short-term outcome for this client? A. The client will use stress-reducing techniques to avoid purging. B. The client will discuss chaos in personal life and be able to verbalize a link to purging. C. The client will gain 2 pounds prior to the next weekly appointment. D. The client will remain free of signs and symptoms of malnutrition and dehydration.

ANS: C The symptoms of anorexia nervosa do not include purging. Correctly written outcomes must be client centered, specific, realistic, measurable, and also include a time frame.

What client information does a nurse need to assess prior to initiating medication therapy with phenelzine (Nardil)? A. The client's understanding of the need for regular blood work B. The client's mood and affect score, using the facility's mood scale C. The client's cognitive ability to understand information about the medication D. The client's access to a support network willing to participate in treatment

ANS: C There are many dietary and medication restrictions when taking Nardil. A client must have the cognitive ability to understand information about the medication and which foods, beverages, and medications to eliminate when taking Nardil.

In the emergency department, a raped client appears calm and exhibits a blunt affect. The client answers a nurse's questions in a monotone using single words. How should the nurse interpret this client's responses? A. The client may be lying about the incident. B. The client may be experiencing a silent rape reaction. C. The client may be demonstrating a controlled response pattern. D. The client may be having a compounded rape reaction.

ANS: C This client is most likely demonstrating a controlled response pattern. In a controlled response pattern, the client's feelings are masked or hidden, and a calm, composed, or subdued affect is seen. In the expressed response pattern, feelings of fear, anger, and anxiety are expressed through crying, sobbing, smiling, restlessness, and tension.

A client diagnosed with bulimia nervosa is to receive fluoxetine (Prozac) by oral solution. The medication is supplied in a 100 mL bottle. The label reads 20 mg/5 mL. The doctor orders 60 mg q day. Which dose of this medication should the nurse dispense? A. 25 mL B. 20 mL C. 15 mL D. 10 mL

ANS: C Twenty mg of Prozac multiplied by three results in the calculated 60 mg daily dose ordered by the physician. Each 5 mL contains 20 mg. Five mL multiplied by three equals the liquid dosage of 15 mL.

A nurse responsible for conducting group therapy on an eating disorder unit schedules the sessions immediately after meals. Which is the rationale for scheduling group therapy at this time? A. To shift the clients' focus from food to psychotherapy B. To prevent the use of maladaptive defense mechanisms C. To promote the processing of anxiety associated with eating D. To focus on weight control mechanisms and food preparation

ANS: C When the nurse schedules group therapy immediately after meals, the nurse is addressing the emotional issues related to eating disorders that must be resolved if these maladaptive responses are to be eliminated.

Sertraline (Zoloft) has been prescribed for a client complaining of poor appetite, fatigue, and anhedonia. Which consideration should the nurse recognize as influencing this prescriptive choice? A. Zoloft is less expensive for the client. B. Zoloft is extremely sedating and will help with sleep disturbances. C. Zoloft has less adverse side effects than other antidepressants. D. Zoloft begins to improve depressive symptoms quickly.

ANS: C Zoloft is a selective serotonin reuptake inhibitor (SSRI) that has a relatively benign side effect profile as compared with other antidepressants.

Which nursing diagnosis should a nurse identify as appropriate when working with a client diagnosed with schizoid personality disorder? A. Altered thought processes R/T increased stress B. Risk for suicide R/T loneliness C. Risk for violence: directed toward others R/T paranoid thinking D. Social isolation R/T inability to relate to others

ANS: D An appropriate nursing diagnosis when working with a client diagnosed with schizoid personality disorder is social isolation R/T inability to relate to others. Clients diagnosed with schizoid personality disorder appear cold, aloof, and indifferent to others. They prefer to work in isolation and are unsociable.

A client exhibits dependency on staff and peers and expresses fear of abandonment. Using Mahler's theory of object relations, which should the nurse expect to note in this client's childhood? A. Lack of fulfillment of basic needs by parental figures B. Absence of the client's maternal figure during symbiosis C. Difficulty establishing trust with the maternal figure D. Inconsistency by the maternal figure during individuation

ANS: D During phase 3 (5 to 36 months) of Margaret Mahler's individuation theory, there should be a strengthening of the ego and an acceptance of "self" with independent ego boundaries. Inconsistency by the maternal figure during individuation may in later years result in feelings of helplessness when the client is alone because of exaggerated fears of being unable to care for self.

A nurse is planning care for a child who is experiencing depression. Which medication is approved by the U.S. Food and Drug Administration (FDA) for the treatment of depression in children and adolescents? A. Paroxetine (Paxil) B. Sertraline (Zoloft) C. Citalopram (Celexa) D. Fluoxetine (Prozac)

ANS: D Fluoxetine (Prozac) is FDA approved for the treatment of depression in children and adolescents. Fluoxetine is a selective serotonin reuptake inhibitor (SSRI) used in the treatment of depression. All antidepressants carry an FDA warning for increased risk of suicide in children and adolescents.

A nurse is caring for four clients taking various medications including imipramine (Tofranil), doxepine (Sinequan), ziprasidone (Geodon), and tranylcypromine (Parnate). The nurse orders a special diet for the client receiving which medication? A. Tofranil B. Senequan C. Geodon D. Parnate

ANS: D Hypertensive crisis occurs in clients receiving monoamine oxidase inhibitor (MAOI) who consume foods or drugs high in tyramine content.

A 20-year-old client and a 60-year-old client have had drunk driving accidents and are both experiencing extreme anxiety. From a psychosocial theory perspective, which of these clients would be predisposed to the diagnosis of adjustment disorder? A. The 60-year-old because of memory deficits B. The 60-year-old because of decreased cognitive processing ability C. The 20-year-old because of limited cognitive experiences D. The 20-year-old because of lack of developmental maturity

ANS: D Research indicates that there is a predisposition to the diagnosis of adjustment disorder when there is limited developmental maturity. By comparison, the 20-year-old does not have the developmental maturity, life experiences, and coping mechanisms that the 60-year-old might possess.

Which client situation should a nurse identify as reflective of the impulsive behavior that is commonly associated with borderline personality disorder? A. As the day shift nurse leaves the unit, the client suddenly hugs the nurse's arm and whispers, "The night nurse is evil. You have to stay." B. As the day shift nurse leaves the unit, the client suddenly hugs the nurse's arm and states, "I will be up all night if you don't stay with me." C. As the day shift nurse leaves the unit, the client suddenly hugs the nurse's arm, yelling, "Please don't go! I can't sleep without you being here." D. As the day shift nurse leaves the unit, the client suddenly shows the nurse a bloody arm and states, "I cut myself because you are leaving me."

ANS: D The client who states, "I cut myself because you are leaving me" reflects impulsive behavior that is commonly associated with the diagnosis of borderline personality disorder. Repetitive, self-mutilative behaviors are common and are generated by feelings of abandonment following separation from significant others.

A client diagnosed with seasonal affective disorder (SAD) states, "I've been feeling 'down' for 3 months. Will I ever feel like myself again?" Which reply by the nurse will best assess this client's symptoms. A. "Have you been diagnosed with any physical disorder within the last 3 months?" B. "Have you experienced any traumatic events that triggered this mood change?" C. "People who have seasonal mood changes often feel better when spring comes." D. "Help me understand what you mean when you say, 'feeling down'?"

ANS: D The nurse is using a clarifying statement in order to gather more details related to this client's mood. The diagnosis of SAD is not associated with a traumatic event.

During an interview, which client statement indicates to a nurse that a potential diagnosis of schizotypal personality disorder should be considered? A. "I really don't have a problem. My family is inflexible, and every relative is out to get me." B. "I am so excited about working with you. Have you noticed my new nail polish: 'Ruby Red Roses'?" C. "I spend all my time tending my bees. I know a whole lot of information about bees." D. "I am getting a message from the beyond that we have been involved with each other in a previous life."

ANS: D The nurse should assess that a client who states that he or she is getting a message from the beyond indicates a potential diagnosis of schizotypal personality disorder. Individuals with schizotypal personality disorder are aloof and isolated and behave in a bland and apathetic manner. The individual experiences magical thinking, ideas of reference, illusions, and depersonalization as part of daily life.

A nurse tells a client that the nursing staff will start alternating weekend shifts. Which response should a nurse identify as characteristic of clients diagnosed with obsessive-compulsive personality disorder? A. "You really don't have to go by that schedule. I'd just stay home sick." B. "There has got to be a hidden agenda behind this schedule change." C. "Who do you think you are? I expect to interact with the same nurse every Saturday." D. "You can't make these kinds of changes! Isn't there a rule that governs this decision?"

ANS: D The nurse should identify that a client diagnosed with obsessive-compulsive personality disorder would have a difficult time accepting change. This disorder is characterized by inflexibility and lack of spontaneity. Individuals diagnosed with this disorder are very serious, formal, over disciplined, perfectionistic, and preoccupied with rules.

A confused client has recently been prescribed sertraline (Zoloft). The client's spouse is taking paroxetine (Paxil). The client presents with restlessness, tachycardia, diaphoresis, and tremors. What complication does a nurse suspect and what could be its possible cause? A. Neuroleptic malignant syndrome caused by ingestion of two different seratonin reuptake inhibitors (SSRIs) B. Neuroleptic malignant syndrome caused by ingestion of an SSRI and a monoamine oxidase inhibitor (MAOI) C. Serotonin syndrome caused by ingestion of an SSRI and an MAOI D. Serotonin syndrome caused by ingestion of two different SSRIs

ANS: D The nurse should suspect that the client is suffering from serotonin syndrome possibly caused by ingesting two different SSRIs (Zoloft and Paxil). Symptoms of serotonin syndrome include confusion, agitation, tachycardia, hypertension, nausea, abdominal pain, myoclonus, muscle rigidity, fever, sweating, and tremor.

A woman describes a history of physical and emotional abuse in intimate relationships. Which additional factor should a nurse suspect? A. The woman may be exhibiting a controlled response pattern. B. The woman may have a history of childhood neglect. C. The woman may be exhibiting codependent characteristics. D. The woman might be a victim of incest.

ANS: D The nurse should suspect that this client might be a victim of incest. Women in abusive relationships often grew up in abusive homes.

A potential Olympic figure skater collapses during practice and is hospitalized for severe malnutrition. Anorexia nervosa is diagnosed. Which client statement best reflects the underlying etiology of this disorder? A. "Skaters need to be thin to improve their daily performance." B. "All the skaters on the team are following an approved 1,200-calorie diet." C. "When I lose skating competitions, I also lose my appetite." D. "I am angry at my mother. I can only get her approval when I win competitions."

ANS: D This client statement reflects the underlying etiology of anorexia nervosa. The client is expressing feelings about family dynamics that may have influenced the development of this disorder. Families who are overprotective and perfectionistic can contribute to a family member's development of anorexia nervosa.

The nurse plans to confront a client about secondary gains related to extreme dependency on spouse. Which nursing statement would be most appropriate? A. "Do you believe dependency issues have been a lifelong concern for you?" B. "Have you noticed any anxiety during times when your husband makes decisions." C. "What do you know about individuals who depend on others for direction?" D. "How have the specifics of your relationship with your spouse benefited you?"

ANS: D When a client goes to excessive lengths to obtain nurturance and support from others, the client is seeking secondary gains. Secondary gains provide clients the support and attention that the client might not otherwise receive.

A client diagnosed with anorexia nervosa stopped eating 5 months ago and lost 25% of total body weight. Which subjective client response would the nurse assess to support this medical diagnosis? A. "I do not use any laxatives or diuretics to lose weight." B. "I am losing lots of hair. It's coming out in handfuls." C. "I know that I am thin, but I refuse to be fat!" D. "I don't know why people are worried. I need to lose this weight."

ANS: D When the client states, "I don't know why people are worried. I need to lose this weight," the client is exhibiting the subjective response of ineffective denial. This client is minimizing symptoms and is unable to admit impact of the disease on life patterns. The client does not perceive personal relevance of symptoms or danger.

Anna has been a widow for 20 years. Her maladaptive grief response to the loss of her dog may be attributed to which of the following? Select all that apply. -unresolved grief over the loss of her husband -loss of several relatives and friends over the last few years -repressed feelings of guilt over the way in which Lucky died -inability to prepare in advance for the loss

All answers are correct. Unresolved grief over the loss of her husband; loss of several relatives and friends of the last few years; repressed feelings of guilt over the way in which Lucky died; inability to prepare in advance for the loss.

A client rates anxiety at 8 out of 10 on a scale of 1 to 10, is restless, and has narrowed perceptions. Which of the following medications would appropriately be prescribed to address these symptoms? Select all that apply. 1. Chlordiazepoxide (Librium). 2. Clonazepam (Klonopin). 3. Lithium carbonate (lithium). 4. Clozapine (Clozaril). 5. Oxazepam (Serax).

An anxiety rating of 8 out of 10, restlessness, and narrowed perceptions all are symptoms of increased levels of anxiety. 1. Chlordiazepoxide (Librium) is a benzodiazepine. Benzodiazepines are classified as antianxiety medications and would be appropriately prescribed to address signs and symptoms of anxiety. 2. Clonazepam (Klonopin) is a benzodiazepine. Benzodiazepines are classified as antianxiety medications and would be appropriately prescribed to address signs and symptoms of anxiety. 5. Oxazepam (Serax) is a benzodiazepine. Benzodiazepines are classified as antianxiety medications and would be appropriately prescribed to address signs and symptoms of anxiety. TEST-TAKING HINT: The test taker first must recognize the signs and symptoms presented in the question as an indication of increased levels of anxiety. Next, the test taker must recognize the medications that address these symptoms. Also, it is common to confuse lithium carbonate (lithium) and Librium and clozapine and clonazepam. To answer this question correctly, the test taker needs to distinguish between medications that are similar in spelling.

An overuse or ineffective use of ego defense mechanisms, which results in a maladaptive response to anxiety, is an example of the ___________________ theory of generalized anxiety disorder development.

An overuse or ineffective use of ego defense mechanisms, which results in a maladaptiveresponse to anxiety, is an example of thepsychodynamic theory of generalized anxiety disorder development. TEST-TAKING HINT: To answer this question correctly, the test taker should review the various theories related to the development of generalized anxiety disorder.

A client is brought to the facility in an agitated state and is admitted to the psychiatric unit for observation and treatment. While putting personal items away, the client talks rapidly and folds and unfolds garments several times. The client can't seem to settle down. Which nursing diagnosis is most applicable at this time?

Anxiety

Minor Tranquilizers (Anti-Anxiety)

CNS depression 2. Anticholinergic These drugs are addicting with prolonged use

All psychiatric drugs have two side effects in common

CNS depression: drowsiness, hypotension, especially when given IM 2. Anticholinergic: Dry mouth, difficulty voiding

PHOBIC DISORDERS Treatment

MEDICATIONS: - Antidepressants (SSRIs) - Beta Blockers (Propranolol); decreases the physical symptoms associated with panic by blocking the effects that adrenaline has on the body. - Benzodiazepines THERAPY: - Cognitive Therapy - Behavior Modification

PTSD Treatment

MEDICATIONS: - Antidepressants (Zoloft, Paxil). - Anxiolytics. - If manifestations include nightmares or insomnia, Minipress has been prescribed. (Normally used to treat HTN, Minipress blocks the brain's response to NE and has been effective in suppressing nightmares.) THERAPY: - Cognitive Therapy - Exposure Therapy

A client is prescribed alprazolam (Xanax) 2 mg bid and 1.5 mg q6h PRN for agitation. The maximum daily dose of alprazolam is 10 mg/d. The client can receive _____ PRN doses of alprazolam within a 24-hour period.

The client can receive 4 PRN doses. Medications are given four times in a 24-hour period when the order reads q6h: 1.5 mg x 4 = 6 mg. The test taker must factor in 2 mg bid = 4 mg. These two dosages together add up to 10 mg, the maximum daily dose of alprazolam (Xanax), and so the client can receive all 4 PRN doses. TEST-TAKING HINT: To answer this question correctly, the test taker must recognize that the timing of standing medication may affect the decision- making process related to administration of PRN medications. In this case, the client would be able to receive all possible doses of PRN medication because the standing and PRN ordered medications together do not exceed the maximum daily dose.

A client is prescribed lorazepam (Ativan) 0.5 mg qid and 1 mg PRN q8h. The maximum daily dose of lorazepam should not exceed 4 mg QD. This client would be able to receive ______ PRN doses as the maximum number of PRN lorazepam doses.

This client should receive 2 PRN doses. The test taker must recognize that medications are given three times in a 24-hour period when the order reads q8h: 1 mg x 3 = 3 mg. The test taker must factor in the 0.5 mg qid = 2 mg. These two dosages together add up to 5 mg, 1 mg above the maximum daily dose of lorazepam (Ativan). The client would be able to receive only two of the three PRN doses of lorazepam. TEST-TAKING HINT: To answer this question correctly, the test taker must recognize that the timing of standing medication may affect the decision-making process related to administration of PRN medications. In this case, although the PRN medication is ordered q8h, and could be given three times, the standing medication dosage limits the PRN to two doses, each at least 8 hours apart.

Phobia

Types 1. Simple phobia: fear of an object or situation 2. Agoraphobia: fear of open or public places 3. Claustrophobia: fear of enclosed or small places C. Defense mechanisms 1. Repression 2. Displacement D. Interventions 1. Gradual desensitization- step by step 2. Behavior modification- rewarding desired behavior 3. Relaxation techniques 4. Do not force confrontation with objects or situation causing phobia 5. Reasoning doesn't work 6. Administer anti-anxiety meds as ordered

After being diagnosed with pyrophobia, the client states, "I believe this started at the age of 7 when I was trapped in a house fire." When examining theories of phobia etiology, this situation would be reflective of ____________ theory.

When examining theories of phobia etiology, this situation would be reflective of learning theory. Some learning theorists believe that fears are conditioned responses, and they are learned by imposing rewards for certain behaviors. In the instance of phobias, when the individual avoids the phobic object, he or she escapes fear, which is a powerful reward. This client has learned that avoiding the stimulus of fire eliminates fear. TEST-TAKING HINT: To answer this question correctly, the test taker needs to review the different theories of the causation of specific phobias.

which of the following dissociative disorders was formerly named multiple personality disorder? -dissociative fugue -dissociative amnesia -dissociative identity disorder -depersonalization disorder

dissociative identity disorder -dissociative identity disorder occurs when the client displays two or more distinct identities or personality staes that recurrently take control of his or her behavior. -dissociative fugue occurs when the clients has episodes of suddenly leaving the home or place of work w/o any explanation. -dissociative amnesia occurs when the client cannot remember important personal information. -depersonalization disorder occurs when the client has a persistent or recurrent feeling of being detached from his or her mental processes or body

A nurse admits an older client with memory loss, confused thinking, and apathy. A psychiatrist suspects depression. What is the rationale for performing a mini mental status exam? -rule out bipolar disorder -rule out schizophrenia -rule out senile dementia -rule out personality disorder

rule out senile dementia

which of the folowing is a priority intervention when dealing with child abuse or neglect?

safety -the first part of treatment for child abuse or neglect is to ensure the childs safety and well-being

which of the following is a priority intervention in the treatment of the client diagnosed w/ PTSD?

safety of the client -promoting the client safety is the priority intervention for the client diagnosed with PTSD. Thenurse continually must assess the clients potential for self-harm or suicide and take action immediately

to preserve evidence from a possible rape with no report of oral sex, the female victim should avoid all of the following except -shower -brushing her teeth -the victim should avoid all of thses activities prior to physical examination -douching

the victim should avoid all of these activities prior to physical examination

The terms "judgment" and "insight" are sometimes used incorrectly. Insight is the ability to:

understand the nature of one's problem or situation.


Related study sets

Nursing health assessment and promotion: Chapter 11: Skin, Hair, and Nails

View Set

Chapter 14. Nervous System: Spinal Cord and Spinal Nerves

View Set

Словник психологічних термінів (А)

View Set

Relativity Certified User Exam - Litigation Support Tip of the Night

View Set

Unit 1, Unit 2a, Unit 2b, Unit 3, Unit 4, Unit 5 -- All Music and Media

View Set

19.2 seismic waves and earths interior

View Set

CS 24o Quiz/Exam Review Questions

View Set